Gyne Smle MCQ 2023

Download as pdf or txt
Download as pdf or txt
You are on page 1of 117

Q1- Female with fishy odor discharge

A. Bacterial vaginosis
B. Candida
C. N gonorrhea

Correct answer: A. bacterial vaginosis.

- Fishy odor discharge is a common symptom associated with bacterial


vaginosis, which is an overgrowth of anaerobic bacteria in the vagina
that disrupts the normal balance of bacteria.
- Candida, on the other hand, is a fungal infection that typically causes
thick, white
discharge and itching.
- N gonorrhea is a sexually transmitted infection that can cause vaginal
discharge, but the discharge is usually yellow or green and may be
accompanied by pain or burning during urination.

Q3- 21 year old patient did cervical screening ( Pap smear)for last
5yrs and it was normal, now she did it again and was normal, when
is the next time for screening ?
A. 6months
B.1year
C.2years
D.3years

Correct answer is: D: 3years

If a 21-year-old patient has had normal cervical screening (Pap smear)


results for the last 5 years, and her most recent result was also normal,
the recommended next time for screening would be in 3 years (D). This
is based on the current
guidelines from the American College of Obstetricians and Gynecolo-
gists (ACOG), which recommend screening for cervical cancer every 3
years for women ages 21 to 29 who have normal Pap smear results.
Dr.Ali Balharith MD, MACP, FRCPC 1
Q2- Patient came with amenorrhea for 6 weeks . Ultrasound shows
empty uterus and sac in right side Hcg 4500 (she is vitally stable)
what’s your diagnosis?
A. Ruptured ectopic pregnancy
B. Ctopic pregnancy
C. Normal pregnancy
D. Threatened abortion

Correct answer is:B.Ctopic pregnancy

- In a patient with amenorrhea for 6 weeks, an empty uterus, a sac in the


right side, and a high hCG level of 4500, the most likely diagnosis is an
ectopic pregnancy (B).
The presence of a sac outside the uterus, along with the high hCG level,
indicates that the fertilized egg has implanted and is growing outside the
uterus, most commonly in the fallopian tube. A ruptured ectopic preg-
nancy (A) would be suspected if the patient had symptoms of internal
bleeding, such as severe abdominal pain, dizziness, or fainting. However,
the patient is reported to be vitally stable, so this is less likely.
A normal pregnancy (C) would be expected to show a gestational sac
within the uterus on ultrasound, which is not the case here. Threatened
abortion (D) is a term used to describe bleeding in early pregnancy, but
it is not applicable in this case since there is no evidence of a viable in-
trauterine pregnancy.

Dr.Ali Balharith MD, MACP, FRCPC 2


Q4- Pregnant with twins prom G3p2 20 years old m. what is the
cause of high risk of bleeding?
B. Maternal age
C. Parity
D. Multiple gestation

Correct answer is D. Multiple gestation

- The high risk of bleeding in a pregnant woman with twins and


premature rupture of membranes (PROM) is most likely due to multiple
gestation (D).
- Multiple gestation pregnancies have a higher risk of complications
including preterm
labor, premature rupture of membranes, and placenta previa, which can
increase the risk of bleeding during pregnancy.
Additionally, the presence of twins can also increase the risk of hemor-
rhage during delivery due to the increased size of the uterus and greater
blood loss during delivery.
- Maternal age (B) and parity (C) may also be risk factors for certain
pregnancy complications, but they are not directly related to the risk of
bleeding in a pregnancy with twins and PROM.

Dr.Ali Balharith MD, MACP, FRCPC 3


Q5- A woman with a left breast mass for 9months. Mammogram:
speculated mass with suspicious microcalcification and axillary
lymph node involvement, BI-RAD V (probable malignancy), next
step?
A. Excisional biopsy
B. Core Biopsy
C. Modified radical mastectomy

Correct answer is: B. Core biopsy.

- In the given scenario, the mammogram results indicate a speculated


mass with suspicious microcalcification and axillary lymph node
involvement, which is classified as BI-RAD V (probable malignancy).
The next step would be to perform a core biopsy to confirm the
diagnosis of breast cancer and determine the type of cancer and the
extent of its spread.
- A core biopsy involves the removal of a small sample of breast tissue
using a needle, which is then examined under a microscope to
determine if cancer cells are present. This procedure is less invasive than
an excisional biopsy or mastectomy and can be performed on an
outpatient basis.
- Once the diagnosis of breast cancer is confirmed, further imaging
studies and tests may be performed to determine the extent of the
cancer and the most appropriate treatment plan, which may include
surgery, radiation therapy, chemotherapy, or a combination of these
treatments.

Dr.Ali Balharith MD, MACP, FRCPC 4


Q6- Female diagnosed myoma in uterus she have bleeding want to
be pregnant?
A.NSAID’s
B.Ocp
C.Progesterone only
D.Combined

Correct answer is: C- Progesterone only.

- In a female diagnosed with myoma in the uterus and is experiencing


bleeding but wants to become pregnant, the most appropriate treatment
option would be Progesterone-only therapy.
- Myomas or fibroids are benign tumors that grow in the uterus.

They can cause symptoms such as heavy menstrual bleeding, pelvic pain,
and pressure.
Progesterone-only therapy can help to control the bleeding associated
with myomas and may also help to shrink the size of the tumors.
- Progesterone-only therapy can be given in the form of oral tablets
injections, or intrauterine devices (IUDs).

It is safe for women who wish to become pregnant as it does not affect
fertility, and it may even help to improve fertility in some cases.

- NSAIDs and combined oral contraceptive pills (OCPs) can also be used
to control bleeding associated with myomas.
However, they are not suitable for women who want to become
pregnant as they can reduce fertility.

Dr.Ali Balharith MD, MACP, FRCPC 5


Q7- Which of the following Occur with ocp?
A.Decrease the risk of ovarian cancer
B.Increase the risk of breast cancer
C. Decrease endometrial cancer
D. Increase risk of ectopic pregnancy

Correct answer is: A. Decrease the risk of ovarian cancer.


- The use of oral contraceptive pills (OCPs) has been associated with
a number of benefits, including a decreased risk of ovarian and
endometrial cancer.
- Studies have shown that long-term use of OCPs (at least five years) can
reduce the risk of ovarian cancer by up to 50%.

This effect is thought to be due to the suppression of ovulation, which


reduces the number of times the ovaries are exposed to potential
carcinogens.

on the other hand, there is evidence that the use of OCPs may be
associated with a slight increase in the risk of breast cancer.
However, this risk appears to be small and is largely limited to women
who use OCPs for more than ten years.

- The use of OCPs does not increase the risk of ectopic pregnancy.

In fact, OCPs are often prescribed to women who are at high risk of
ectopic pregnancy, as they can help to regulate the menstrual cycle and
reduce the risk of ovulation occurring in the fallopian tubes.

- In summary, the use of OCPs has been associated with a decreased risk
of ovarian and endometrial cancer, a slight increase in the risk of breast
cancer (with long-term use), and no increase in the risk of ectopic
pregnancy.

Dr.Ali Balharith MD, MACP, FRCPC 6


Q8- Pregnant with asthma what happens during pregnancy?
A.increase RR
B. Increase tidal volume
C. Increase resdual volume
D. Increase functional resdual capacity

Correct answer is: D. Increase functional resdual capacity

- During pregnancy, there are several changes that occur in the respira-
tory system
including in women with asthma. These changes include:
-A. Increased respiratory rate (RR): During pregnancy, there is an in-
crease in oxygen
demand, which causes an increase in RR to meet the increased metabolic
demands of the mother and fetus.
- B. Increased tidal volume: Tidal volume is the amount of air that is
breathed in and out during a normal breath. During pregnancy, tidal vol-
ume increases to meet the increased oxygen demand.
- C. Increased residual volume: Residual volume is the amount of air that
remains in the lungs after a maximal exhalation. During pregnancy, re-
sidual volume increases due to the relaxation of the smooth muscles in
the airways, which can make it more difficult to breathe for women with
asthma.
- D. Increased functional residual capacity: Functional residual capaci-
ty is the amount of air that remains in the lungs at the end of a normal
breath. During pregnancy
functional residual capacity decreases due to the upward displacement
of the diaphragm by the growing uterus, which can also make it more
difficult to breathe for women with asthma.

Dr.Ali Balharith MD, MACP, FRCPC 7


Q9- Meig’s syndrome how to treat?
A. Surgery and chemo
B. Surgey and radio
C. Radio

Correct answer is: A. surgery and chemo

- Meig’s syndrome is a rare condition that is characterized by the pres-


ence of a benign ovarian tumor (usually a fibroma) along with pleural
effusion and ascites. The treatment of Meig’s syndrome involves surgical
removal of the ovarian tumor, which is usually curative.
- However, in some cases, chemotherapy is also recommended to prevent
recurrence of the tumor. This is particularly true if the tumor is large or
if there is evidence of metastasis. Chemotherapy can help to shrink the
tumor and prevent the growth of any cancer cells that may be present.
- Radiotherapy is not typically used as a treatment for Meig’s syndrome.
While it may be used in the treatment of some types of ovarian tumors,
it is not considered to be a first-line treatment for fibromas.

Q261- Patient with irregular period and and infertility husband se-
men analysis in normal what is the diagnosis?
A. Ovulatory dysfunction
B. Endometriosis
C. Falopian tube problem

Correct answer is: A. ovulatory dysfunction

- Irregular periods can be a sign of ovulatory dysfunction, which means


that the ovaries are not releasing eggs regularly. This can lead to infer-
tility as there may not be an egg available for fertilization during the
woman’s fertile window. In cases where the husband’s semen analysis is
normal, it is less likely that the infertility is due to male factor issues.
- Endometriosis and fallopian tube problems can also cause infertility,
but they are typically associated with other symptoms such as pelvic
Dr.Ali Balharith MD, MACP, FRCPC 8
pain, heavy or painful periods, or painful intercourse. In this scenario,
those symptoms are not mentioned, making it less likely that the diag-
nosis is endometriosis or fallopian tube problems.

Q10- Patient diagnosed with subserosal 2 cm fibroid asymptomatic


but worried about fertility what is most appropriate?
A. myomectomy
B. Reassure
C. Hysterectomy

Correct answer is: B. Reassure

- Subserosal fibroids are located on the outer surface of the uterus and
typically do not affect fertility unless they are very large or located in a
specific area that can interfere with implantation or blood flow to the
uterus. A 2 cm fibroid is considered relatively small and is unlikely to in-
terfere with fertility.
- Myomectomy, which involves the surgical removal of the fibroid while
preserving the uterus, may be considered in cases where the fibroid is
causing significant symptoms or if the patient is trying to conceive and
the fibroid is in a location that may interfere with implantation or blood
flow. However, since the patient in this scenario is asymptomatic and
the fibroid is small, myomectomy is not necessary.
- Hysterectomy, which involves the removal of the uterus, is only con-
sidered as a last resort in cases where other treatments have failed or
if there is a significant risk of cancer or other serious conditions. It is
not appropriate in this scenario as the patient is asymptomatic and con-
cerned about fertility.
- The patient should be advised that the fibroid is unlikely to affect her
fertility and that she can continue to try to conceive naturally.
If she experiences any symptoms in the future, she should seek medical
attention.

Dr.Ali Balharith MD, MACP, FRCPC 9


Q11- Pregnant at 34week in clinic presented with high bp 140/75 , no
proteinuria what tk give
A. L dopa
B. Hydrlazine
C. Nifidipine
D. Mg sulphate

Correct answer is: C. Nifidipine

- Nifedipine is a calcium channel blocker that is commonly used to treat


hypertension in pregnancy. It is considered safe and effective for both
the mother and fetus.

- L-dopa (levodopa) is a medication used to treat Parkinson’s disease and


is not indicated for the treatment of hypertension in pregnancy.

- Hydralazine is a direct-acting vasodilator that is used to treat hyper-


tension in pregnancy, but it is not the preferred first-line drug due to its
potential side effects such as
tachycardia and headaches.

- Magnesium sulfate is a medication used for the prevention and treat-


ment of seizures in women with preeclampsia or eclampsia, but it is not
indicated for the treatment of
hypertension alone.

Dr.Ali Balharith MD, MACP, FRCPC 10


Q12- Research for group of cardiac patient half will recieve new drug
and half will receive placebo , what is there right
A. Withdral at anytime
B. Explaining and knowing the research instrument
C. Explaining the results or the procedure can’t remember

Correct answer is: B. Explaining and knowing the research instru-


ment

- If a group of cardiac patients is involved in a research study, where half


of them will
receive a new drug and half will receive a placebo, their rights include “b.
Explaining and knowing the research instrument.”
- Patients have the right to be fully informed about the research study,
including the
purpose, potential benefits, risks, and procedures involved. They must
be given the
opportunity to ask questions, clarify doubts, and understand the nature
of their
participation before giving their informed consent.
- Patients also have the right to know that they can withdraw from the
study at any time without any penalty or negative consequences. They
should be informed about the
confidentiality of their data and how it will be used in the study.

Dr.Ali Balharith MD, MACP, FRCPC 11


Q13- Pregnant lady came to hospital with abroken arm after her hus-
band beat her, whats the most likely time to increase violence?
A. After retirement
B. After family visit
C. Holidays
D. At work

Correct answer is: C.Holidays

- Based on research, the most likely time for an increase in domestic vio-
lence is C. Holidays.
- The holiday season can be a stressful time for many people, with added
financial pressures, family obligations, and other stressors.
- This stress can exacerbate existing tensions in relationships and
increase the likelihood of domestic violence.
- Additionally, alcohol consumption may increase during the holiday sea-
son, which can also contribute to violent behavior.
- While domestic violence can occur at any time, studies have shown
that rates of domestic violence tend to increase during holiday periods,
such as Christmas, New Year’s Eve, and Thanksgiving.
- However, it is important to note that domestic violence can occur at
any time, and all instances of domestic violence should be taken
seriously and addressed promptly.

Q14- Pregnancy on tube 5cm patient lives far away absolute contra-
indications of methotrexate
A. Distance from hospital?
B. Obesity
C. Ultrasound
D. BCG 3500

Correct answer is: C.Ultrasound


- The absolute contraindication of methotrexate in the case of a pregnan-
cy on the tube measuring 5cm is a positive BCG greater than 3500.
Dr.Ali Balharith MD, MACP, FRCPC 12
Q15- Pregnant with emesis from 2 weeks ? Hb low MCV high ‘ which
vit deficiency?
A. B12
B. Folate

Correct answer is: B.Folate

- The low hemoglobin and high MCV in a pregnant woman with emesis
for two weeks suggest a possible folate deficiency. Therefore, option B
(Folate) is the correct answer.
- Folate (vitamin B9) is essential for the synthesis of DNA and red blood
cells, and it is
particularly important during pregnancy for the growth and develop-
ment of the fetus.
Folate deficiency can lead to megaloblastic anemia, which is character-
ized by a high mean corpuscular volume (MCV) and low hemoglobin lev-
els.
- Pregnant women are at an increased risk of folate deficiency due to the
increased
demand for folate by the developing fetus and placenta. Nausea and vom-
iting during pregnancy can also contribute to folate deficiency by reduc-
ing oral intake and increasing losses through vomiting.
- Vitamin B12 deficiency (option A) can also cause megaloblastic anemia,
but it is less likely to be the cause in this case because the high MCV is
suggestive of folate deficiency. Vitamin B12 deficiency is more common-
ly associated with neurological symptoms and is less common in preg-
nant women who consume adequate amounts of animal products.
- In summary, in a pregnant woman with emesis for two weeks, a low
hemoglobin and high MCV suggest a possible folate deficiency.

Dr.Ali Balharith MD, MACP, FRCPC 13


Q16- Ectopic pregnancy, the husband refused for termination, con-
sent ?
A. From patient
B. From the patient and ask husband about it

Correct answer is:A. From patient

- In the case of an ectopic pregnancy where the husband refuses termi-


nation, the consent for the procedure must be obtained from the patient.
- In medical decision-making, the patient’s autonomy and right to make
decisions about their own body are paramount. Even if the husband dis-
agrees with the termination of the pregnancy, the final decision should
rest with the patient. It is important to have
a discussion with the husband and explain the risks and potential conse-
quences of not terminating the ectopic pregnancy. However, the deci-
sion to proceed with or without the husband’s consent should ultimately
be based on the patient’s wishes. In situations where the patient is un-
able to provide informed consent, such as in cases of mental incapacity
or emergency situations, the healthcare team may need to seek consent
from the next of kin or a legal representative. However, in this case, the
patient is capable of providing informed consent, and her wishes should
be respected. In summary, in the case of an ectopic pregnancy where
the husband refuses termination, the consent for the procedure must be
obtained from the patient.

Dr.Ali Balharith MD, MACP, FRCPC 14


Q17- Ectopic with bhcg plateau for 7 days?
A. Repeat after one week
B. Repeat after 48 hrs
C. Observe

Correct answer is:B. Repeat after 48 hrs

- In the case of an ectopic pregnancy with a plateau in beta-hCG levels for


7 days, the appropriate management would be to repeat the beta-hCG
levels after 48 hours.
Therefore, option B (Repeat after 48 hrs) is the correct answer.
- Beta-hCG levels are commonly used to monitor the progression of
ectopic pregnancy.
- A plateau in beta-hCG levels indicates a lack of resolution of the ectopic
pregnancy and can be a sign of persistent trophoblastic tissue or a non-
viable pregnancy.
- In cases where beta-hCG levels plateau, repeat testing after a short
interval can help to determine whether the levels are continuing to rise
or have started to decline.
- Waiting for one week (option A) may delay diagnosis and treatment,
and observing
(option C) is not appropriate as ectopic pregnancies can be life-
threatening.
- Therefore, the best course of action would be to repeat the beta-hCG
levels after 48 hours.
If the levels are continuing to rise or have not started to decline, further
intervention, such as methotrexate treatment or surgery, may be neces-
sary.
- In summary, in the case of an ectopic pregnancy with a plateau in
beta-hCG levels for 7 days, the appropriate management would be to
repeat the beta-hCG levels after 48 hours to determine the next steps in
management.

Dr.Ali Balharith MD, MACP, FRCPC 15


Q18- Pregnant in 28 weeeks, increased vaginal discharge . Speculum
examination revealed pooling in fornices. Dx?
A. PROM
B. Normal physiological vaginal discharge

Correct answer is:B. Normal physiological vaginal discharge.

- Increased vaginal discharge is a common symptom during pregnancy


due to hormonal changes and increased blood flow to the genital area.
Speculum examination may show pooling of discharge in the fornices,
which is a normal finding in pregnancy.
It is important to ensure that there is no associated odor, itching, or irri-
tation, which may indicate an infection.
- PROM (Premature Rupture of Membranes) is a condition in which the
amniotic sac
ruptures before the onset of labor, usually before the 37th week of preg-
nancy.
- However, PROM is typically associated with a gush of fluid from the
vagina, and the fluid may continue to leak afterwards.
- The presence of vaginal discharge alone is unlikely to be indicative of
PROM.

Dr.Ali Balharith MD, MACP, FRCPC 16


Q19- Definitive tx of endometriosis, after failure of all treatments.
A. Hysterectomy and bilateral oorphorectomy
B. D&C

Correct answer is: A. Hysterectomy and bilateral oophorectomy.

- Endometriosis is a condition where endometrial-like tissue grows out-


side the uterus and can lead to the development of chronic pelvic pain,
painful periods, and infertility.
- Treatment for endometriosis typically starts with medical therapy,
which includes hormonal therapy and pain management, followed by
minimally invasive surgical interventions such as laparoscopy and abla-
tion/resection of the lesions.
- If these treatments fail to provide relief or the endometriosis is severe
or recurrent, then the definitive treatment option is usually a hysterecto-
my (removal of the uterus) and bilateral oophorectomy (removal of both
ovaries).
- Since endometriosis is estrogen-dependent, the removal of the ovaries
will eliminate the source of estrogen, which can help prevent the growth
and spread of endometrial tissue.
- This procedure is usually reserved for women who have completed
childbearing and have failed all other treatments.
- D&C (Dilatation and Curettage) is a diagnostic and therapeutic proce-
dure that involves the removal of the uterine lining.
- However, it is not an appropriate treatment option for endometriosis.

Dr.Ali Balharith MD, MACP, FRCPC 17


Q20- 20 years old, c/o right abdominal pain and mass. Pregnancy
test negative, normal vitals. Dx?
A. Ovarian torsion
B. Ectopic pregnancy
C. Cystic rupture

Correct answer is:A. Ovarian torsion.

- Abdominal pain and mass in a young female can be due to various un-
derlying pathologies.
- However, given the normal pregnancy test and the absence of signs of
shock, ectopic pregnancy is less likely. Cystic rupture may present with
abdominal pain and tenderness, but a palpable mass is not a typical find-
ing.
- Ovarian torsion occurs when the ovary twists around its blood supply,
leading to ischemia and infarction of the ovarian tissue.
- It can present with sudden onset abdominal pain, lower quadrant or
right abdominal pain, and often a palpable mass.
- Since torsion can potentially lead to the loss of the ovary and fallopian
tube, prompt diagnosis and management are necessary.
- Diagnostic tests for ovarian torsion include pelvic ultrasound, which
may show an enlarged ovary and a “whirlpool” sign with decreased blood
flow, and diagnostic laparoscopy may be required for diagnosis and fur-
ther management.
B. Ectopic pregnancy:
- Ectopic pregnancy occurs when the fertilized egg implants outside the
uterus, commonly in the fallopian tube. It can cause abdominal pain, pel-
vic pain, vaginal bleeding, and a positive pregnancy test. Ectopic preg-
nancy can also cause signs of shock, such as tachycardia, dizziness, and
hypotension. However, in this case, since the pregnancy test is negative
and there were normal vitals, the possibility of an ectopic pregnancy is
less likely.
C. Cystic rupture:
- Ovarian cysts are fluid-filled sacs that commonly occur during a wom-
Dr.Ali Balharith MD, MACP, FRCPC 18
an’s reproductive years. They can be asymptomatic or may cause abdom-
inal pain, bloating, and irregular menstruation.
- The rupture or torsion of an ovarian cyst can cause sudden onset, se-
vere lower abdominal pain. A palpable mass is not a typical feature of a
ruptured cyst.
- A ruptured cyst can also lead to internal bleeding and can cause symp-
toms such as lightheadedness, dizziness, and low blood pressure.
- However, a cyst rupture is less likely in this case because a palpable
mass was reported.

Q21- Scenario of pregnant lady in labour surgeon decided to c/s she


need anther opinion
A. Consent her for discharge
B. Father wish
C. Do c/s anyway

Correct answer is:C.Do c-section anyway.

- If a surgeon has determined that a cesarean section (c-section) is med-


ically necessary for the safety of the mother and/or baby, then it should
be performed as soon as possible. It is important to make sure that the
patient and significant others have received an
appropriate explanation of the risks and benefits of the procedure, and
that the patient has provided informed consent.
- While taking the father’s opinion or considering the patient’s wish is
important, it should not delay necessary medical treatment. If the c-sec-
tion is determined to be the safest
option for the mother and/or baby, then it should be performed as soon
as possible
without any unnecessary delay.
-

Dr.Ali Balharith MD, MACP, FRCPC 19


Q22- Woman is 10 weeks gestation and isn’t immune to Rubella?
A. Give her rubella vaccine now
B. Give rubella vaccine at 28 weeks
C. Give rubella post partum
D. Don’t give rubella

Correct answer is:A. Give her rubella vaccine now

- When a woman is 10 weeks pregnant and is not immune to Rubella, it is


important to take appropriate measures to prevent the risk of congenital
Rubella syndrome in the
developing fetus. Therefore, the correct course of action would be to
give the Rubella
vaccine after confirming pregnancy status and ensuring that the patient
is not pregnant or does not become pregnant for at least 28 days after
vaccination. Option A is the correct answer.
- Rubella is a viral illness that can cause serious complications in pregnan-
cy, including miscarriage, stillbirth, and congenital Rubella syndrome. It
is important to ensure that all women of childbearing age are immune to
Rubella before they become pregnant.
- If a woman is found to be non-immune to Rubella during pregnancy,
vaccination can be considered to prevent infection and reduce the risk
of congenital Rubella syndrome.
However, the vaccine should be given after confirming pregnancy status
and ensuring that the patient is not pregnant or does not become preg-
nant for at least 28 days after vaccination.
- Option B, which suggests giving the Rubella vaccine at 28 weeks, is not
the correct course of action as it does not address the risk of congenital
Rubella syndrome during the first trimester of pregnancy.
- Option C, which suggests giving Rubella postpartum, is also not the
correct course of action as it does not address the risk of congenital Ru-
bella syndrome during pregnancy.
- Option D, which suggests not giving the Rubella vaccine, is not appro-
priate as it would put the developing fetus at risk of congenital Rubella
Dr.Ali Balharith MD, MACP, FRCPC 20
syndrome.
- In conclusion, when a woman is 10 weeks pregnant and is not immune
to Rubella, the correct course of action would be to give the Rubella vac-
cine after confirming pregnancy status and ensuring that the patient is
not pregnant or does not become pregnant for at least 28 days after vac-
cination. Option A is the correct answer.

Q23- Mono mono twins. When did separation happen?


A. 0-72 hours
B. 3-7 days
C. 8-12 days
D. > 12 days

Correct answer is:D. > 12 days

- Monoamniotic-monochorionic (MoMo) twins are identical twins who


share the same amniotic sac and placenta.
- The separation of MoMo twins occurs later than the separation of
diamniotic twins, who have their own amniotic sacs. The correct answer
is D. >12 days.
- MoMo twins are rare and are associated with a higher risk of
complications, including cord entanglement and compression, which
can lead to fetal death or neurological damage.
- The timing of separation is important as it can affect the outcomes for
both twins.
- Studies have shown that the separation of MoMo twins occurs around
14-15 days after fertilization, which is later than the separation of
diamniotic twins. This means that MoMo twins have a higher risk of
complications due to the longer period of time they spend in close
proximity to each other in the same amniotic sac.
- Option A, which suggests that separation occurs 0-72 hours after
fertilization, is incorrect as this is too early for MoMo twins to separate.
- Option B, which suggests that separation occurs 3-7 days after
fertilization, is also incorrect as this is still too early for MoMo twins to
Dr.Ali Balharith MD, MACP, FRCPC 21
separate.
- Option C, which suggests that separation occurs 8-12 days after
fertilization, is also incorrect as this is still too early for MoMo twins to
separate.
- In conclusion, the correct answer is D. >12 days, as MoMo twins
typically separate around 14-15 days after fertilization.

Q24- 30 years old pregnant 28 week gestation with contraction that


have been occurring every 6 minutes, the patient has history of
long-standing DM type 1, which has been complicated had prenatal
courses. Vaginal exam demonstrates approximately 1 cm of
cervical dilation and mild cervical effacement. The patient is started
on management sulfate, antibiotic and betamethasone. What is the
primary concern of corticosteroid administration in the patient?
A. Increased maternal insulin requirement
B. Maternal infection
C. Fetal hyperglycemia
D. None of the above

Correct answer is: C. Fetal hyperglycemia

- The primary concern of corticosteroid administration in this patient is


fetal hyperglycemia because the patient has long-standing DM type 1.
- Corticosteroids can cause fetal hyperglycemia as they increase
maternal glucose levels, which crosses the placenta and increases fetal
glucose levels.
- This can be dangerous in diabetic mothers, who may already have high
fetal glucose levels.
- Proper glucose monitoring is important in such patients to prevent
adverse outcomes such as fetal macrosomia, neonatal hypoglycemia, and
neonatal intensive care unit admissions.
- Maternal insulin requirement may need to be adjusted during
corticosteroids administration, but fetal hyperglycemia is the primary
concern.
Dr.Ali Balharith MD, MACP, FRCPC 22
- Maternal infection is a concern in any patient, but it is not specific to
corticosteroid administration.

Q25- Female newly married was on ocps even before marriage, now
wanting to continue but her husband want her to stop ?
A. Patient consent is enough
B. Both should be consented
C. Her consent should be taken after discussion with her husband

Correct answer is:A.Patient consent is enough

- The decision to continue or discontinue oral contraceptive pills (OCPs)


should ultimately be made by the patient after a thorough discussion
with her healthcare provider.
- While it is important to understand the concerns of the patient’s hus-
band, ultimately it is the patient who is taking the medication and may
experience side effects or other issues related to its use.
- Therefore, the patient’s consent is enough to continue or discontinue
OCPs.
- However, healthcare providers may encourage the involvement of the
patient’s partner or spouse in discussions related to family planning and
contraceptive choices.

Dr.Ali Balharith MD, MACP, FRCPC 23


Q26- Pregnant with uncontrolled GDM What’s complication
expected to happen her newborn
A. Hypergylcemia
B. Low Hb
C. Low erythropoietin
D. Respiratory distress syndrome

Correct answer is:A. Hyperglycemia

- Gestational diabetes mellitus (GDM) is a condition characterized by


high blood glucose levels that emerge during pregnancy.
- If uncontrolled, the high glucose levels in the mother’s blood can cross
the placenta and cause hyperglycemia in the developing fetus.
- This can lead to several complications in the newborn, including
macrosomia (large birth weight), hypoglycemia (low blood glucose
levels), respiratory distress syndrome, and neonatal intensive care unit
admissions. Therefore, in this case, the newborn of a mother with
uncontrolled GDM is expected to have hyperglycemia as a result of
exposure to high blood glucose levels in utero.
- This can cause numerous complications post-delivery, such as
respiratory distress syndrome, hypoglycemia, and neonatal seizures.
- It is important for the mother with GDM to receive appropriate
medical care and management to help prevent complications in the
newborn. This includes regular monitoring of blood glucose levels and
maintaining glucose levels within a normal range through diet, exercise,
and medication as necessary.
- Proper management can help reduce the risk of complications to both
the mother and the newborn.

Dr.Ali Balharith MD, MACP, FRCPC 24


Q27-23year old woman w/ right knee pain 3wks ago. not been
relieved by rest, Ice, NSAIDs. right knee pain that is worse with
activity, frequent catching of the joint, persistent edema. believes
she injured her knee while hiking three weeks ago. While hiking
she felt pain in her knee and swelling. tenderness along the medial
joint line. Anterior and posterior drawer testing is negative. Flex-
ion and extension of the knee with external rotation of the tibia
reproduces her pain. What structure injured?
A. Medial meniscus
B. Anterior cruciate ligament
C. Medial collateral ligament
D. Posterior cruciate ligament

Correct answer is:A. Medial meniscus

- In this case, the patient presented with right knee pain that has not
been relieved by rest, ice, or NSAIDs. Additionally, the pain has
worsened with activity, frequent catching of the joint, and persistent
edema.
- These symptoms suggest an injury to a knee structure, likely the medial
meniscus.
- The medial meniscus is a common site of injury, especially in patients
who have a history of twisting or pivoting injuries to the knee, such as
in the case of hiking.
- On examination, the patient has tenderness along the medial joint line,
and reproduces her pain with flexion and extension of the knee with ex-
ternal rotation of the tibia.
- These findings are consistent with an injury to the medial meniscus,
which may cause pain and causing catching or locking in the knee joint.
- Anterior and posterior drawer testing is negative, which makes it less
likely that the patient has an ACL or PCL injury.
- Similarly, the absence of significant valgus or varus stress on examina-
tion makes it less likely that the patient has an MCL or LCL injury.
- Therefore, in this case, the most likely structure injured is the medial
Dr.Ali Balharith MD, MACP, FRCPC 25
meniscus, which is a common site of injury in patients presenting with
these symptoms.

Q28-Old women came with heavy bleeding hysteroscopy was done


,Hysteroscopy was ( look at the pic What is the diagnosis ?
A. Adenomyosis
B. Endometrial polyps
C. Endometriosis

Correct answer is: B. Endometrial polyps.

- Endometrial polyps are benign growths that develop on the lining of


the uterus.
- They are often small and round, but can sometimes be larger and
irregular in shape.
- Endometrial polyps can cause heavy menstrual bleeding, spotting
between periods, and infertility.
- The treatment for endometrial polyps is to remove them surgically.
This can be done with a hysteroscopy, which is a procedure that uses a
thin, lighted tube to view the inside of the uterus.
- The polyps can then be removed with a small surgical tool.
- The prognosis for endometrial polyps is generally good.
- Once the polyps are removed, the symptoms usually go away However,
there is a small risk of the polyps coming back.
- Adenomyosis is a condition in which the tissue that lines the uterus
(the endometrium) grows into the muscle wall of the uterus.
- Adenomyosis can cause heavy menstrual bleeding, pain, and infertility.
- Endometriosis is a condition in which the tissue that lines the uterus
(the endometrium) grows outside the uterus, usually on the ovaries, fal-
lopian tubes, or other organs in the pelvic area.
- Endometriosis can cause pain, infertility, and other problems.
- The hysteroscopy image shows a polyp on the lining of the uterus.
- The polyp is small and round, and it is not attached to the muscle wall
of the uterus. This is consistent with an endometrial polyp.
Dr.Ali Balharith MD, MACP, FRCPC 26
Q29-38 weeks gesteginal she has not felt the fetal movements since
yesterday. Investigations confirm fetal demise. how will you know
the Causes?
A. Placental sampling.
B. Cord sampling.
C. Fetus sampling

Correct answer is: A. placental sampling.

- Placental sampling is the most common way to determine the cause of


fetal demise.
- The placenta is the organ that provides oxygen and nutrients to the fe-
tus, and it can also contain clues about the cause of death.
- Cord sampling is another option, but it is not as common as placental
sampling.
- The umbilical cord is the connection between the mother and the fe-
tus, and it can also contain clues about the cause of death.
- Fetus sampling is the least common option, and it is only used in very
specific cases.
- This is because it can be difficult to obtain a sample from the fetus, and
it can also be difficult to interpret the results.
- The most common causes of fetal demise are:
* Placental abruption: This is when the placenta separates from the wall
of the uterus before the baby is born.
* Umbilical cord prolapse: This is when the umbilical cord comes out of
the uterus before the baby is born.
* Birth defects: These are problems that are present at birth.
* Infections: These can be caused by bacteria, viruses, or parasites.
* Maternal health problems: These can include diabetes, high blood pres-
sure, or heart disease.
If a woman has a fetal demise, she should see her doctor to determine
the cause.
The doctor will likely do a physical exam and order tests, such as blood
tests and an ultrasound.
Dr.Ali Balharith MD, MACP, FRCPC 27
The doctor may also recommend a placental sampling or a cord sampling

Q30- 39 years old at 13 weeks gestation for chorionic villus sampling


after an elevated first trimester aneuploidy screen. fetal karyotype
of 47.xx +16. What is the next step in the management of this
pregnancy?
A. Offer amniocentesis
B. Schedule a fetal anatomical survey: plan for regular growth
ultrasound and non-stress testing during pregnancy.
C. Advise termination of this pregnancy due to lethal karyotype
results
D. Offer parental karyotype testing.

Correct answer is: C. Advise termination of this pregnancy due to


lethal karyotype results.

- Trisomy 16 is a chromosomal abnormality that occurs when there are


three copies of chromosome 16 instead of the normal two.
- It is the most common cause of miscarriage in the first trimester.
- Trisomy 16 is also associated with a number of birth defects, including
heart defects, neural tube defects, and cleft lip and palate.
- Babies with trisomy 16 usually do not survive to birth.
If they do survive, they have severe developmental delays and
intellectual disabilities.
- There is no cure for trisomy 16. Treatment is supportive and aimed at
relieving symptoms.
- Parents of babies with trisomy 16 should be offered genetic counseling
to help them understand the condition and its implications.
- If a woman is found to have a fetus with trisomy 16, she may be offered
the option of terminating the pregnancy.
- This is a difficult decision, and parents should be given time to consider
all of their options.
A. Offer amniocentesis
Dr.Ali Balharith MD, MACP, FRCPC 28
Amniocentesis is a procedure in which a sample of amniotic fluid is
taken from the uterus to test for genetic abnormalities. It is not
necessary in this case, as the karyotype of the fetus is already known.
B. Schedule a fetal anatomical survey: plan for regular growth ultrasound
and non-stress testing during pregnancy.
A fetal anatomical survey is a detailed ultrasound scan of the fetus. It is
not necessary in this case, as the fetus is not expected to survive to birth.
D. Offer parental karyotype testing.
Parental karyotype testing is not necessary in this case, as the karyotype
of the fetus is already known.

Q31- 39 years old at 13 weeks gestation for chorionic villus sampling


after an elevated first trimester aneuploidy screen. fetal karyotype
of 47.xx +16. What is the next step in the management of this preg-
nancy?
A. Offer amniocentesis
B. Schedule a fetal anatomical survey: plan for regular growth
ultrasound and non-stress testing during pregnancy.
C. Advise termination of this pregnancy due to lethal karyotype
results
D. Offer parental karyotype testing.

The correct answer is C. Advise termination of this pregnancy due


to lethal karyotype results.

Trisomy 16 is a chromosomal abnormality that is incompatible with life.


The vast majority of pregnancies affected by trisomy 16 will end in mis-
carriage or stillbirth.
In the rare cases where a baby is born with trisomy 16, they will have se-
vere health problems and will likely die within a few days or weeks.
Given the poor prognosis for babies with trisomy 16, it is important to
counsel parents about the option of terminating the pregnancy.
This is a difficult decision for any parent to make, but it is important to
remember that trisomy 16 is a lethal condition and that there is no cure.
Dr.Ali Balharith MD, MACP, FRCPC 29
If parents choose to continue with the pregnancy, they should be aware
that their baby will likely have severe health problems and will need in-
tensive medical care.
They should also be prepared for the possibility that their baby may not
survive.
It is also important to offer parental karyotype testing to determine if
the trisomy 16 is due to a de novo mutation or if one of the parents is a
carrier of a balanced translocation.
If one of the parents is a carrier of a balanced translocation, there is an
increased risk of having another child with a chromosomal abnormality.

Q32- AT 26week gestation presents due an abnormal glucose


tolerance test. One week prior, she was given 50g of oral glucose and
demonstrated a venous plasma glucose level of 156mg/dL 1 hour lat-
er, what is the next step in management?
A. Administer an oral 3 hour 100g glucose dose
B. Advise following the American diabetic association diet plan.
C. Order a fetal US examination
D. Begin insulin treatment

Correct answer is: A. Administer an oral 3 hour 100g glucose dose.

The patient has an abnormal glucose tolerance test, which is defined as a


plasma glucose level of 140 mg/dL or higher 1 hour after a 50g oral
glucose load.
This is consistent with gestational diabetes mellitus (GDM), which is a
type of diabetes that occurs during pregnancy.
The next step in management is to confirm the diagnosis of GDM by
performing a 3-hour 100g oral glucose tolerance test.
This test is more sensitive and specific than the 1-hour 50g test.
If the patient has GDM, she will need to be treated with insulin or other
medications to control her blood sugar levels.
The patient should also be advised to follow a healthy diet and exercise
regularly.
Dr.Ali Balharith MD, MACP, FRCPC 30
These lifestyle changes can help to control blood sugar levels and reduce
the risk of complications from GDM.
The patient should also be monitored closely throughout her pregnancy.
This includes regular blood sugar checks, ultrasounds, and fetal
monitoring.
GDM can increase the risk of complications for both the mother and the
baby, so it is important to identify and treat it early.

Q33- 74 due to vaginal spotting and discharge, mass coming out of


her vagina, having difficulty defecating, increased straining causes
the mass to increase in size. The patient has hypertension, type 2
DM and hemoglobin A1c was 11.8.
had 5 spontaneous vaginal deliveries. Oxygen saturation is 89% on
room air. pelvic exam reveals the cervix at the level of the vaginal
introitus. The vaginal mucosa is thin with multiple areas of exco-
riation and erosion, US reveals a thin endometrial stripe, manage-
ment of the patient?
A. Pessary fitting
B. Surgical repair
C. Endometrial biopsy
D. Pelvic floor muscle exercise

Correct answer is: A. Pessary fitting.

The patient is a 74-year-old woman with a history of hypertension, type


2 diabetes mellitus, and vaginal spotting and discharge. She has had five
spontaneous vaginal deliveries. Her oxygen saturation is 89% on room
air. Pelvic exam reveals the cervix at the level of the vaginal introitus.
The vaginal mucosa is thin with multiple areas of excoriation and ero-
sion. US reveals a thin endometrial stripe.
These findings are consistent with a diagnosis of pelvic organ prolapse
(POP). POP is a condition in which the pelvic organs, such as the uterus,
bladder, and rectum, descend from their normal position. It is a common
condition, especially in women who have had multiple vaginal deliveries.
Dr.Ali Balharith MD, MACP, FRCPC 31
The management of POP depends on the severity of the prolapse and
the patient’s symptoms. For mild cases, pessary fitting may be an op-
tion. Pessaries are devices that are inserted into the vagina to support
the pelvic organs. They are a non-surgical option that can be effective in
treating POP.
If the prolapse is more severe or if the patient is not a good candidate for
pessary fitting, surgical repair may be necessary. Surgical repair can be
performed laparoscopically or vaginally.
The patient in this case should be referred to a urogynecology specialist
for further evaluation and management.
The other answer choices are not as likely to be the best management
for this patient. Endometrial biopsy is a procedure that is used to obtain
a sample of tissue from the lining of the uterus. It is used to diagnose
conditions such as endometrial cancer. However, there is no evidence
that the patient in this case has endometrial cancer.
Pelvic floor muscle exercise (Kegel exercises) can be helpful in
strengthening the muscles that support the pelvic organs.
However, Kegel exercises are not likely to be effective in treating the
patient’s POP.

Dr.Ali Balharith MD, MACP, FRCPC 32


Q34- 34 year old antenatal check up at the 24th week, her uterine
size corresponds to her dates. and her OGTT IS NORMAL. URINE
EXAMAINTION SHOWS glycosuria by glucose oxidase strip testing.
How would you advise her?
A. She should be reassured that this is a normal phenomenon in
pregnancy
B. She should be advised to control her diet to prevent hyperglyce-
mia, which is the cause of her glycosuria, after a week, she will need
a repeat urine testing.
C. She has an increased risk of diabetes in pregnancy, which needs
to be ruled out by a repeat OGTT
D. Her normal blood glucose with the pressure of glycosuria implies
renal damage and will require renal function testing

Correct answer is: C. she has an increased risk of diabetes in preg-


nancy, which needs to be ruled out by a repeat OGTT.

The patient is a 34-year-old woman at 24 weeks gestation. Her uterine


size corresponds to her dates. Her OGTT is normal. Urine examination
shows glycosuria by glucose oxidase strip testing.
The presence of glycosuria in a pregnant woman is a risk factor for ges-
tational diabetes mellitus (GDM). GDM is a type of diabetes that occurs
during pregnancy. It is caused by the body’s inability to use glucose ef-
fectively. GDM can have serious consequences for both the mother and
the baby.
The patient in this case should be advised to have a repeat OGTT to rule
out GDM. If she is diagnosed with GDM, she will need to be monitored
closely and treated with diet, exercise, and medication.
The other answer choices are not as likely to be the best advice for this
patient.
A. Reassurance that glycosuria is a normal phenomenon in pregnancy is
not accurate. While glycosuria can be seen in normal pregnancies, it is
more likely to be seen in women with GDM.
B. Advising the patient to control her diet to prevent hyperglycemia is
Dr.Ali Balharith MD, MACP, FRCPC 33
not a specific enough recommendation. The patient should be advised to
follow a healthy diet and to monitor her blood sugar levels.
D. Renal damage is not a likely explanation for the patient’s glycosuria.
The patient should be evaluated for GDM, but renal function testing is
not necessary at this time.

Q35- 25year old w/ metabolic vaginal discharge, four weeks ago the
patient had a forceps assisted vaginal birth with a prolonged second
stage of labor. The delivery was complicated by a third-degree
laceration, postpartum endometritis, which was treated. During
the pregnancy, the patient was diagnosed with vulvovaginal
candidiasis. Her history includes irritable bowel syndrome.
examination shows small, red, velvety area on the posterior vaginal
wall with foul smelling brown discharge. Which of the following is
the most likely diagnosis in this patient?
A. Rectovaginal fistula
B. Incisional abscess
C. Anal incontinence
D. Retained vaginal gauze

Correct answer is: A. rectovaginal fistula.

A rectovaginal fistula is an abnormal connection between the rectum


and the vagina. It can be caused by childbirth, surgery, or infection.
The patient in this case had a forceps assisted vaginal birth with
a prolonged second stage of labor. She also had a third-degree laceration,
which is a tear that extends through the entire thickness of the vaginal
wall.
These factors increase the risk of developing a rectovaginal fistula.
The patient’s symptoms of a foul-smelling brown discharge and a small,
red, velvety area on the posterior vaginal wall are consistent with
a rectovaginal fistula. The discharge is likely coming from the rectum
and the velvety area is likely granulation tissue, which is a type of tissue
Dr.Ali Balharith MD, MACP, FRCPC 34
that forms in response to inflammation.

The other answer choices are not as likely to be the diagnosis in this pa-
tient.

* Incisional abscess is a collection of pus that forms at the site of an in-


cision. This is not a common complication of childbirth and the patient
does not have any other symptoms that would suggest an abscess.
* Anal incontinence is the involuntary loss of stool. This is not a com-
mon complication of childbirth and the patient does not have any other
symptoms that would suggest anal incontinence.
* Retained vaginal gauze is a piece of gauze that is left in the vagina after
surgery. This is a rare complication of surgery and the patient does not
have any other symptoms that would suggest retained vaginal gauze.
The patient should be referred to a gynecologist for further evaluation
and treatment.

Q36- When prescribe oral contraceptive for, which of the following


will prompt serious consideration of whether to recommend an-
other method of contraception?
A. Smoking
B. Parity
C. Diabetes
D. Serum HDL

Correct answer is: A. Smoking.

When prescribing oral contraceptives, the presence of certain risk fac-


tors may prompt serious consideration of whether to recommend an-
other method of contraception. Smoking is a well-known risk factor for
cardiovascular disease and stroke, and the use of combined oral contra-
ceptives (COCs) in smokers is associated with an increased risk of these
complications. Therefore, women who smoke should be advised to quit
smoking and, if they choose to use COCs, should use a low-dose formu-
Dr.Ali Balharith MD, MACP, FRCPC 35
lation.
Parity (B) refers to the number of live births a woman has had and is not
a contraindication to the use of oral contraceptives.
Diabetes (C) is not a contraindication to the use of oral contraceptives,
but women with diabetes may be at increased risk of cardiovascular dis-
ease and should be advised to use a low-dose formulation.
Serum HDL (D) is a measure of the level of “good” cholesterol in the
blood, and it is not a contraindication to the use of oral contraceptives.
However, women with low HDL levels may be at increased risk of car-
diovascular disease and should be advised to use a low-dose formulation

Q37- 16 year old woman, recently become sexually active. what is


recommendation regarding cervical screening?
A. Begin 3 year interval cervical cancer screenings via pap smear at
age 21.
B. Begin 2 year interval cervical cancer screening via pap smear
today
C. Offer the HPV vaccine so that pap smears can be avoided
D. Begin 2 year cervical cancer screenings via pap smear at age 21

Correct answer is: A. Begin 3 year interval cervical cancer screen-


ings via pap smear at age 21.

Current guidelines from the American College of Obstetricians and Gy-


necologists (ACOG) recommend that cervical cancer screening begin at
age 21, regardless of sexual activity. Women under the age of 21 are at low
risk for cervical cancer and may be harmed by unnecessary testing and
treatment.
For women aged 21 to 29, ACOG recommends cervical cancer screening
with a Pap test every three years. HPV testing is not recommended for
this age group unless it is needed to follow up an abnormal Pap test re-
sult.
The HPV vaccine is recommended for all boys and girls at age 11 or 12, be-
fore they become sexually active. The vaccine can help prevent the types
Dr.Ali Balharith MD, MACP, FRCPC 36
of HPV that are most likely to cause cervical cancer. However, vaccina-
tion does not eliminate the need for cervical cancer screening.
Therefore, in this case, the recommendation would be to begin cervical
cancer screening with a Pap test at age 21, with subsequent screenings
every three years. The HPV vaccine should also be offered to help pre-
vent cervical cancer.

Q38- 28 years old pregnant healthy, no health problems w/ new mur-


mur. currently in the second trimester.
murmur found 1 week ago, asymptomatic, Examination is low-
pitched, early diastolic murmur that is best heard at the apex and
bilateral symmetric mild lower extremity edema. cause of murmur
and peripheral edema?
A. Normal physiological adaptations
B. Chronic obstructive pulmonary disease
C. CHF
D. Mitral valve dysfunction

Correct answer is: A. Normal physiological adaptations.

The patient is a 28-year-old pregnant woman in the second trimester with


no prior health problems. She has a new murmur that is low-pitched and
early diastolic, and is best heard at the apex. She also has bilateral sym-
metric mild lower extremity edema.
These findings are all consistent with normal physiological adaptations
to pregnancy. During pregnancy, the heart has to work harder to pump
blood around the body. This can cause a murmur to develop. The in-
creased blood volume can also cause fluid to build up in the legs, causing
edema.
The patient is asymptomatic and has no other signs or symptoms of
heart disease. Therefore, the most likely cause of her murmur and ede-
ma is normal physiological adaptations to pregnancy.
Chronic obstructive pulmonary disease (COPD) is a lung disease that
causes shortness of breath. It is not a common cause of murmurs or ede-
Dr.Ali Balharith MD, MACP, FRCPC 37
ma in pregnancy.
Congestive heart failure (CHF) is a condition in which the heart cannot
pump blood effectively. It can cause shortness of breath, fatigue, and
edema. However, it is not a common cause of murmurs in pregnancy.
Mitral valve dysfunction is a condition in which the mitral valve does not
close properly. It can cause a murmur, but it is not a common cause of
edema in pregnancy.

Q39- A 29 year old woman who can’t get pregnant she has been men-
struating every other month and has a milky discharge from the
nipples, prolactin level 334 and pituitary adenoma 0.5cm in MRI,
which therapeutic options is most appropriate at this time?
A. trial of a dopamine agonist
B. transsphenoidal pituitary surgery
C. no treatment
D. radiation therapy

Correct answer is: A.Trial of a dopamine agonist.

The patient’s presentation is consistent with hyperprolactinemia, which


is an excess of prolactin hormone in the blood. This can be caused by
a pituitary adenoma, as seen in this patient’s MRI. Hyperprolactinemia
can cause menstrual irregularities, infertility, and milky discharge from
the nipples.
The first-line treatment for hyperprolactinemia is a dopamine agonist,
such as bromocriptine or cabergoline. These medications work by stimu-
lating dopamine receptors in the pituitary gland, which reduces the pro-
duction of prolactin.
In this case, a trial of a dopamine agonist is the most appropriate thera-
peutic option at this time. The patient’s prolactin level is elevated, and
her symptoms are consistent with hyperprolactinemia. A dopamine ag-
onist can help to reduce her prolactin level, normalize her menstrual
cycle, and improve her chances of getting pregnant.
Transsphenoidal pituitary surgery (B) may be considered if medical ther-
Dr.Ali Balharith MD, MACP, FRCPC 38
apy is not effective or if the patient has a large pituitary adenoma that is
causing visual or neurological symptoms. However, this is a more inva-
sive option and is typically reserved for cases where medical therapy has
failed.
No treatment (C) is not appropriate in this case, as the patient’s hyper-
prolactinemia is causing symptoms and may be contributing to her in-
fertility.
Radiation therapy (D) is not typically used as a first-line treatment for
hyperprolactinemia, as it has a delayed onset of action and can cause
long-term complications. It may be considered in cases where medical
and surgical therapy have failed.

Q40- Pregnant at 8 weeks of gestation with history or spontaneous


fetal loss at 20 week. What is the most appropriate action to do?
A. cerclage at 14 - 16 weeks
‎‫‏‬B. Cervical cerclage now

Correct answer is: A. cerclage at 14 - 16 weeks.

A cerclage is a surgical procedure that is used to close the cervix. The


cervix is the lower part of the uterus that opens into the vagina. A cer-
clage is used to prevent preterm birth.
In this case, the patient is pregnant at 8 weeks of gestation with a history
of spontaneous fetal loss at 20 weeks. This suggests that the patient has a
weak cervix. A cerclage can help to prevent preterm birth by closing the
cervix and keeping it closed.
A cerclage is usually performed between 14 and 16 weeks of gestation.
This is because the cervix is usually at its weakest between 14 and 16
weeks of gestation.
A cerclage is a safe and effective procedure that can help to prevent
preterm birth. It is important to note that a cerclage is not a guarantee
that the patient will not have a preterm birth. However, it can help to
reduce the risk of preterm birth.
Cervical cerclage now is not the most appropriate action to do. It is be-
Dr.Ali Balharith MD, MACP, FRCPC 39
cause the cervix is usually at its weakest between 14 and 16 weeks of
gestation. A cerclage is usually performed between 14 and 16 weeks of
gestation.

Q41- Healthy young lady just delivered a baby 4 months ago. Howev-
er, she tells you that she does not want to get pregnant for the next
2 years. What method of contraception will you recommend?
A. Combined OCP
B. Depo Provera injection
C. Vaginal ring
D. Transdermal patch

Correct answer is: B. Depo Provera injection.

Depo-Provera is a progestin-only contraceptive injection that is given ev-


ery 3 months. It is a very effective form of birth control, with a failure
rate of less than 1%. Depo-Provera is also safe for most women to use,
including breastfeeding women.
The combined oral contraceptive pill (OCP) is another effective form of
birth control, but it is not recommended for breastfeeding women. The
OCP contains both estrogen and progestin, and estrogen can decrease
milk supply.
The vaginal ring and the transdermal patch are also effective forms of
birth control, but they are not as effective as Depo-Provera. The vaginal
ring is a small, flexible ring that is inserted into the vagina.
The transdermal patch is a small, thin patch that is placed on the skin.
Depo-Provera is the best method of contraception for this patient
because it is effective, safe, and convenient. It is also a good choice for
breastfeeding women because it does not decrease milk supply.

Dr.Ali Balharith MD, MACP, FRCPC 40


Q42-Mono mono twins. When did separation happen?
A. 0-72 hours
B. 3-7 days
C. 8-12 days
D. > 12 days

Correct answer is: D. >12 days.

Monoamniotic-monochorionic (Mono Mono) twins share the same am-


niotic sac and placenta. The timing of separation in Mono Mono twins is
not well defined, but it is believed to occur after the embryonic disk has
implanted in the chorion and started to form the placenta, which occurs
around day 9-12 after fertilization.
Therefore, the most correct answer is D. >12 days because Mono Mono
twins cannot be separated before the embryonic disk has implanted in
the chorion. This is usually after the first week of fertilization.

Q43Pregnant with cottage cheese discharge how to treat:


A. Topical antifungal
B. Oral antifuncal

Correct answer is: A. topical antifungal.

Cottage cheese discharge is a common symptom of yeast infection, which


is caused by an overgrowth of the fungus Candida albicans.
Yeast infections are more common during pregnancy because of hor-
monal changes.
Topical antifungal medications are the first-line treatment for yeast
infections during pregnancy. These medications are available over-the
counter in the form of creams, ointments, or suppositories. They are safe
to use during pregnancy and are effective in treating yeast infections.
Oral antifungal medications are not recommended for use during preg-
nancy unless the topical medications are not effective. Oral antifungal
medications can cause side effects, such as nausea, vomiting, and diar-
Dr.Ali Balharith MD, MACP, FRCPC 41
rhea. They can also interact with other medications.
If you are pregnant and have cottage cheese discharge, you should see
your doctor. Your doctor can diagnose the infection and prescribe the
appropriate treatment.

Q44- Female in 20’s with multiple fibroids, best way to preserve her
fertility
A. Myomectomy
B. OCP
C. Progesterone

Correct answer is: A. myomectomy.

Myomectomy is a surgical procedure to remove uterine fibroids. It is the


best way to preserve fertility in a woman with multiple fibroids.
Oral contraceptive pills (OCPs) and progesterone are hormonal treat-
ments that can help to shrink fibroids. However, they are not as effective
as myomectomy at preserving fertility.
Myomectomy is a safe and effective procedure. The risks of myomecto-
my include bleeding, infection, and damage to the uterus. However, the
risks are generally low, and the benefits of myomectomy outweigh the
risks.
Myomectomy is the best way to preserve fertility in a woman with mul-
tiple fibroids. It is a safe and effective procedure.

Dr.Ali Balharith MD, MACP, FRCPC 42


Q45- A female pregnant at 7weeks of gestation, presented with vag-
inal spotting. U/S:
Fetal heart is positive. PV examination shows a closed OS. What are
the possible
diagnoses?
A. Missed abortion
B. Threatened abortion
C. Inevitable abortion

Correct answer is: B. threatened abortion.

A threatened abortion is a condition in which a pregnant woman has vag-


inal bleeding and/or cramping, but the cervix is still closed. This means
that the pregnancy is still viable, but there is a risk of miscarriage.
A missed abortion is a condition in which the fetus has died, but the
body has not yet expelled it. This can happen for a variety of reasons, in-
cluding chromosomal abnormalities, infections, and placental abruption.
An inevitable abortion is a condition in which the cervix has begun to
dilate and the fetus is being expelled. This is a miscarriage that cannot be
stopped.
In this case, the patient is 7 weeks pregnant and has vaginal spotting. The
ultrasound shows a positive fetal heart rate, which means that the fetus
is still alive. The pelvic examination shows a closed cervix, which means
that the pregnancy is still viable. Therefore, the most likely diagnosis is a
threatened abortion.
The patient should be advised to rest and avoid strenuous activity. She
should also be given a follow-up appointment to monitor the pregnancy.

Dr.Ali Balharith MD, MACP, FRCPC 43


Q46-Young female pregnant in her 28th week, pregnancy test was
done at home. She presented with mild bleeding loss of fetal
movement. Shedenied any passage of tissue or abdominal pain.
Transvaginal US wasdone and showed pregnancy of 18 weeks and
no heartbeat. What is yourdiagnosis?
A. Fetal demise
B. Missed Abortion
C. Incomplete abortion

Correct answer is: A. fetal demise.

Fetal demise is the death of a fetus in utero. It can occur at any time
during pregnancy, but is most common in the third trimester. The cause
of fetal demise is often unknown, but it can be due to a variety of factors,
including placental abruption, umbilical cord accidents, and maternal
infections.
Missed abortion is a condition in which the fetus has died, but the body
has not yet expelled it. This can happen for a variety of reasons
including chromosomal abnormalities, infections, and placental
abruption.
Incomplete abortion is a condition in which the fetus has been expelled,
but some of the products of conception (such as the placenta) have
remained in the uterus. This can lead to infection and bleeding.
In this case, the patient is 28 weeks pregnant and has had a positive preg-
nancy test. She has also had mild bleeding and loss of fetal movement.
The ultrasound shows a pregnancy of 18 weeks and no heartbeat. This
indicates that the fetus has died. Therefore, the most likely diagnosis is
fetal demise.
The patient should be offered counseling and support. She may also need
to be hospitalized for monitoring and treatment.

Dr.Ali Balharith MD, MACP, FRCPC 44


Q47- Pregnancy have ejection systolic murmur radiating to carotid
artery
A. Aortic stenosis
B. Mitral stenosis
C. Aortic regurgitation
D. Pregnancy related murmur

Correct answer is: D. Pregnancy related murmur.

Pregnancy related murmurs are common and are usually benign. They
are caused by the increased blood flow and changes in the heart that
occur during pregnancy. The murmurs are usually heard best over the
left side of the chest and may radiate to the neck or carotid arteries.
Aortic stenosis is a narrowing of the aortic valve, which can cause
a systolic murmur. Mitral stenosis is a narrowing of the mitral valve,
which can cause a diastolic murmur. Aortic regurgitation is a backflow of
blood from the aorta into the left ventricle, which can cause a diastolic
murmur.
Pregnancy related murmurs are usually not associated with any
symptoms. However, if the murmur is loud or associated with other
symptoms, such as chest pain or shortness of breath, it should be evalu-
ated by a doctor.

Q41-A patient in 8wks GA with mild bleeding and on exam, cervix


was close , What is the type of abortion?
A. Threatened
B. Inevitable
C. Incomplete

Correct answer is: A. Threatened abortion.

A threatened abortion is a condition in which a pregnant woman has vag-


inal bleeding and/or cramping, but the cervix is still closed. This means
Dr.Ali Balharith MD, MACP, FRCPC 45
that the pregnancy is still viable, but there is a risk of miscarriage.
An inevitable abortion is a condition in which the cervix has begun to
dilate and the fetus is being expelled. This is a miscarriage that cannot be
stopped.
An incomplete abortion is a condition in which the fetus has been
expelled, but some of the products of conception (such as the placenta)
have remained in the uterus. This can lead to infection and bleeding.
In this case, the patient is 8 weeks pregnant and has mild bleeding. The
cervix is closed, which means that the pregnancy is still viable.
Therefore, the most likely diagnosis is a threatened abortion.
The patient should be advised to rest and avoid strenuous activity. She
should also be given a follow-up appointment to monitor the pregnancy

Q48- Pregnant woman, high gravida (forgot the number) and known
previous caesareans, has metastatic cancer. Doctor advised for ter-
mination of pregnancy but her husband is refusing?
A. you need both consent from her and husband
B. Only need her consent
C. Only her consent and she should inform her husband
D. Consent of representative

Correct answer is: B. Only need her consent.

In the United States, a woman has the right to make her own decisions
about her healthcare, including whether or not to terminate a pregnan-
cy. This right is protected by the Constitution and by state law. The hus-
band’s consent is not required.
However, it is important to have a conversation with the husband about
the situation. He may be feeling scared, angry, or confused. It is import-
ant to listen to his concerns and to answer his questions.
The doctor can also offer support and resources to the husband.
In some cases, the husband may be able to provide emotional support to
the woman during the pregnancy termination process.
In other cases, the woman may prefer to have a support person who is
Dr.Ali Balharith MD, MACP, FRCPC 46
not her husband. It is important to respect the woman’s wishes.
If the woman is not sure what she wants to do, the doctor can offer her
more information about the risks and benefits of continuing the
pregnancy or terminating it. The doctor can also refer the woman to
a counselor or therapist who can help her make a decision.
Ultimately, the decision of whether or not to terminate a pregnancy is
up to the woman. The doctor can provide information and support, but
the woman has the right to make her own decision.

Q49- Increasingly heavy vaginal bleeding and fatigue over the past
6 months, fm hx for myocardial infarction, Abdominal exam shows
firm, palpable mobile mass just blew the umbilicus, on pelvic ex-
amination, and there is a moderate amount of old, clotted blood
coming from the cervical OS, hematocrit of 24% and hemoglobin
is low, which diagnostic test is most cost effective in confirming a
diagnosis?
A. Transvaginal US
B. Hysterosalpingogram
C. Abdominal and pelvic CT
D. Diagnostic laparoscopy

Correct answer is: A. transvaginal US

Transvaginal ultrasound is a non-invasive procedure that uses sound


waves to create images of the uterus, ovaries, and fallopian tubes.
It is a very sensitive test that can be used to detect a variety of
conditions, including fibroids, polyps, and tumors.
The patient in this case has a history of heavy vaginal bleeding and
fatigue. She also has a firm, palpable mobile mass just below the
umbilicus. These symptoms are consistent with a variety of conditions,
including fibroids, polyps, and tumors.
Transvaginal ultrasound is a cost-effective way to evaluate these
conditions.
It is a non-invasive procedure that does not require sedation or
Dr.Ali Balharith MD, MACP, FRCPC 47
anesthesia.
It is also a relatively quick procedure that can be performed in the office.
The other options are more expensive and invasive.

* *Hysterosalpingogram:* This is a procedure that uses X-rays to


examine the uterus and fallopian tubes. It is a more invasive procedure
than
transvaginal ultrasound and requires sedation or anesthesia.
* *Abdominal and pelvic CT:* This is a medical imaging test that uses
X-rays to create detailed images of the abdomen and pelvis. It is a more
expensive and invasive procedure than transvaginal ultrasound.
* *Diagnostic laparoscopy:* This is a surgical procedure that allows the
doctor to look inside the abdomen and pelvis. It is the most invasive pro-
cedure of the options listed and requires general anesthesia.

If the transvaginal ultrasound is inconclusive, the doctor may recom-


mend a hysterosalpingogram, abdominal and pelvic CT, or diagnostic
laparoscopy.
However, transvaginal ultrasound is a good first-line test in this case.

Q50- 58 years old F, Menarche was at age 14 and menopause occurred


4 years ago, A cousin on the paternal side of her family died of breast
cancer, which of the following is the greatest risk factor for breast
cancer in this patient?
A. Family History of breast cancer
B. Age at first pregnancy
C. Duration of breast feeding
D. Chronological age

Correct answer is: A. Family history of breast cancer.

Having a family history of breast cancer is the greatest risk factor for
developing breast cancer.
Dr.Ali Balharith MD, MACP, FRCPC 48
The risk of developing breast cancer increases if a woman has a first
degree relative (mother, sister, or daughter) who has been diagnosed
with breast cancer.
The risk is even higher if the relative was diagnosed with breast cancer
before the age of 50.
The other options are not as significant risk factors for breast cancer.
* *Age at first pregnancy:* Women who have their first child at an older
age are at an increased risk of developing breast cancer.
However, the risk is still relatively low, and it is not as significant as
a family history of breast cancer.
* *Duration of breastfeeding:* Breastfeeding can slightly lower the risk
of developing breast cancer. However, the effect is small, and it is not as
significant as a family history of breast cancer.
* *Chronological age:* The risk of developing breast cancer increases
with age. However, the risk is still relatively low in women under the age
of 50.
A family history of breast cancer is a much more significant risk factor
than chronological age.
The patient in this case is a 58-year-old woman with a family history of
breast cancer.
Her cousin on the paternal side of her family died of breast cancer.
This is a significant risk factor, and she should be evaluated by a doctor
for early detection of breast cancer.

Dr.Ali Balharith MD, MACP, FRCPC 49


Q51- 27 years old at 18 weeks gestation comes for a routine prenatal
visit. The patient feels well and accompanied by her mother as her
husband was unable to get off work, us shows a cephalic
singleton with severe growth restriction, bilateral choroid plexus
cysts, clenched fists and a large ventricular septal defect
appropriate initial statement by the physician?
A. There are some things about your ultrasounds that i need to
discuss with you, is it okays to do that now?”
B. Would it be okay if your mother stepped out of the room so we
can talk about your results”
C. Let’s schedule a follow up visit to discuss the results when your
husband can be there here with you”
D. “could i speak to you in private about my concerns with your
ultrasound findings?”

Correct answer is:B. Would it be okay if your mother stepped out of


the room so we can talk about your results

It is important to allow the patient to have privacy when receiving bad


news. The patient may want to discuss her options with her husband or
another trusted family member. It is also important to allow the patient
to express her emotions without feeling judged or embarrassed.
The physician should start by asking the patient if she would like her
mother to step out of the room. This shows respect for the patient’s
privacy and allows her to feel comfortable discussing her options. The
physician should then explain the results of the ultrasound in a clear and
concise way. The physician should be prepared to answer the patient’s
questions and provide her with support.

The following are some additional tips for breaking bad news:
* Be honest and direct.
* Avoid using euphemisms or jargon.
* Allow the patient time to process the news.
* Be patient and understanding.
Dr.Ali Balharith MD, MACP, FRCPC 50
* Offer emotional support.
* Provide information about resources that can help.

Q52- A pregnant lady with swelling in the left leg, diagnosed as DVT,
which of the following is the best management?
A. Duplex and heparin
B. Duplex and heparin and IVC filter
C. Warfarin

Correct answer is:A. Duplex and heparin

Option B (Duplex and heparin and IVC filter) may be considered in cer-
tain cases where there is a high risk of pulmonary embolism, but it is not
the first-line management. Warfarin is contraindicated in pregnancy due
to its teratogenic effects.

Q53- 28 weak still birth with congenital anamoly , how will you know
the Causes ?
A. Placental saming.
B. Cord sampling.
C. Fetus sampling

Correct answer is:C. Fetus sampling

Fetal sampling is the most likely to help determine the cause of a still-
birth with congenital anomaly. The placenta and umbilical cord can also
be sampled, but they are less likely to provide as much information.
Fetal sampling can be performed in a number of ways, including:
* *Amniocentesis:* This involves inserting a needle through the mother’s
abdomen and into the amniotic sac to remove a small amount of amni-
otic fluid. The fluid contains cells from the fetus, which can be examined
for genetic abnormalities or structural defects.
* *Chorionic villus sampling (CVS):* This involves inserting a needle
Dr.Ali Balharith MD, MACP, FRCPC 51
through the mother’s abdomen and into the placenta to remove a small
amount of tissue from the placenta. The tissue contains cells from the
fetus, which can be examined for genetic abnormalities or structural de-
fects.
* *Fetal blood sampling:* This involves inserting a needle through the
mother’s abdomen and into the umbilical cord to remove a small amount
of blood from the fetus. The blood can be examined for genetic abnor-
malities or infections.
Fetal sampling is a relatively safe procedure, but it does carry some
risks, such as infection and bleeding. The risks of fetal sampling should
be weighed against the potential benefits of obtaining information that
could help prevent future stillbirths.

Q54- Young female presented with heavy vaginal bleeding has been
diagnosed with uterine fibroid. Her hemoglobin is 7. What is the
most appropriate next step?
A. Myomectomy
B. Correct the anemia
C. Combined Oral Contraceptive Pills
D. blood transfusion

Correct answer is: B. Correct the anemia

The most appropriate next step for a young female presented with heavy
vaginal bleeding, diagnosed with uterine fibroid, and hemoglobin of 7
is to *correct the anemia*. This can be done with a blood transfusion or
iron supplements. Once the anemia is corrected, the patient can then be
evaluated for further treatment of the uterine fibroids.
Here are the reasons why the other options are not the best choice:
* *Myomectomy:* This is a surgical procedure to remove uterine fibroids.
It is not the best option for a patient with severe anemia, as it would re-
quire general anesthesia and could lead to further blood loss.
* *Combined oral contraceptive pills (COCPs):* COCPs can help to con-
trol heavy menstrual bleeding, but they are not effective in treating ane-
Dr.Ali Balharith MD, MACP, FRCPC 52
mia. They can also increase the risk of blood clots, which is a concern for
a patient with severe anemia.
* *Blood transfusion:* This is the most immediate way to correct the
anemia. However, it is not a long-term solution. The patient will need to
be evaluated for further treatment of the uterine fibroids to prevent the
anemia from recurring.

Q55- Pregnant with high tsh and constipation, cold intolerance


what you give...
A. Propylthiouracel
B. Thyroxin

Correct answer is:B. Thyroxin

If a pregnant woman has a high TSH level, constipation, and cold intol-
erance, the most appropriate medication to give would be thyroxine (B).
Thyroxine is a synthetic form of the thyroid hormone that can help reg-
ulate the body’s metabolism. In pregnant women, hypothyroidism (low
thyroid hormone levels) can cause a range of complications, including
miscarriage, preterm delivery, and developmental delays in the fetus.
Propylthiouracil (A) is a medication used to treat hyperthyroidism (over-
active thyroid), which is the opposite of hypothyroidism. It is not appro-
priate for a pregnant woman with high TSH levels.
Therefore, the correct answer is B. Thyroxine.

Q56- Severe Dysmenorrhea with infertility for 6 years, Dx?


A. Endometriosis
B. Adenomyosis

Correct answer is:A. Endometriosis

Severe dysmenorrhea (painful periods) with infertility for 6 years is a


common feature of endometriosis (A).
Endometriosis is a condition where the tissue that normally lines the
Dr.Ali Balharith MD, MACP, FRCPC 53
inside of the uterus grows outside of it, causing inflammation, scarring,
and adhesions. This can result in painful periods, chronic pelvic pain,
and infertility.
Adenomyosis (B) is a similar condition where the tissue that normally
lines the inside of the uterus grows into the muscular wall of the uterus,
causing pain, heavy bleeding, and enlargement of the uterus. While ade-
nomyosis can also cause painful periods and infertility, it is typically not
associated with severe dysmenorrhea to the same degree as endometri-
osis.

Q57- G1 p0 at 28 weeks gestation presented with membrane rupture


24h ago, what is your management ?
A. Observation
B. Induction of labor (IOL)
C. Emergency cesarean section
D. Amnioinfusion

Correct answer is: B. Induction of labor (IOL)

The most likely management options would be:


* *Observation:* The woman may be monitored closely for signs of
preterm labor or fetal distress. If no complications develop, the woman
may be able to continue the pregnancy for several more weeks.
* *Induction of labor (IOL):* If the woman is at 34 weeks gestation or
more, the doctor may induce labor to deliver the baby. IOL is usually
done with medication, such as oxytocin.
* *Emergency cesarean section:* If the woman or the fetus is in distress,
the doctor may perform an emergency cesarean section to deliver the
baby.
* *Amnioinfusion:* Amnioinfusion is a procedure in which fluid is in-
jected into the amniotic sac. This can help to prevent the umbilical cord
from becoming compressed and can also help to improve the oxygen-
ation of the fetus.
Dr.Ali Balharith MD, MACP, FRCPC 54
Here are some of the factors that the doctor will consider when making
a decision about the best management option:
* *The woman’s overall health:* The doctor will want to make sure that
the woman is healthy enough to carry the pregnancy to term.
* *The gestational age of the fetus:*
The doctor will want to make sure that the fetus is developed enough to
survive outside the womb.
* *The presence of any complications:* The doctor will want to make
sure that there are no complications that could put the woman or the
fetus at risk.
The doctor will also want to consider the woman’s wishes and preferenc-
es.
The woman may want to try to prolong the pregnancy for as long as pos-
sible, or she may want to deliver the baby sooner.

Q58- Pregnant in her 15 GA came for Follow up, expected to have :


A. Decrease Hemoglobin 20%
B. Decrease hemoglobin 40%
C. Decrease plasma volume 20%
D. Decrease plasma volume 40%

Correct answer is:C. Decrease plasma volume 20%.

During pregnancy, there is a physiological increase in plasma volume.


This increase is greater than the increase in red blood cell mass, which
leads to a dilution of the blood and a decrease in hemoglobin concentra-
tion. The decrease in hemoglobin concentration is typically about 10%
at 20 weeks gestation and 15% at term. However, the decrease in plasma
volume is typically only about 20% at 20 weeks gestation and 30% at term.
Therefore, a pregnant woman who is at 15 weeks gestation is expected
to have a decrease in plasma volume of about 20%. This decrease in plas-
ma volume can lead to symptoms such as lightheadedness, fatigue, and
shortness of breath.
In addition to the decrease in plasma volume, there are other physiolog-
Dr.Ali Balharith MD, MACP, FRCPC 55
ical changes that occur in the blood during pregnancy. These changes
include:
* Increase in white blood cell count
* Increase in platelet count
* Decrease in hematocrit
* Decrease in mean corpuscular hemoglobin concentration
* Decrease in mean corpuscular hemoglobin
These changes are necessary to support the growth and development of
the fetus.

Q59-Woman with dysuria, allergic to sulfa and penicillin, what will


you give her ?
A. Tmp- sulfa
B. Nitro
C Cefalaxin
D. Ampicillin

Correct answer is:B. Nitro

In a patient who is allergic to both sulfa and penicillin and presents with
dysuria, the recommended antibiotic treatment option is nitrofurantoin,
option B. Nitrofurantoin is an antibiotic that is commonly used to treat
urinary tract infections and is not related to the sulfa or penicillin anti-
biotic classes. However, it is important to confirm the diagnosis of a uri-
nary tract infection through urine culture and sensitivity testing before
initiating antibiotic treatment. The choice of antibiotic treatment may
also depend on the severity of the infection, patient factors, and local
resistance patterns.

Dr.Ali Balharith MD, MACP, FRCPC 56


Q60- Woman delivered after she delivered the placenta she lost
1000cc of blood, uteres is firm she is alert and stable, what’s the Dx ?
A. Atony
B. Coagolapathy
C Retained products of placenta
D. Genital late

Correct answer is:A. Atony

In a woman who has delivered and lost 1000cc of blood after delivering
the placenta, with a firm uterus, and who is alert and stable, the likely
diagnosis is uterine atony, option A.
Uterine atony is a common cause of postpartum hemorrhage and is
defined as the inability of the uterus to contract effectively after
delivery.
This can lead to excessive bleeding and requires prompt management,
including uterine massage, administration of uterotonic medications,
and potentially surgical interventions such as a hysterectomy or uterine
artery embolization. Coagulopathy, retained products of conception, and
genital tract trauma are other potential causes of postpartum
hemorrhage that should also be considered and ruled out through appro-
priate evaluation and management

Dr.Ali Balharith MD, MACP, FRCPC 57


Q61- Pregnant known diabetic 40weeks un eventful pregnancy,
sudden fetal death, fetus was 2.6kg no obvious anomaly, placental
sample showed thrombi, what’s the cause of the fetal death:
A. Diabetes
B. Growth restriction
C. Placental abnormality
D. hypoglycaemia

Correct answer is:C. Placental abnormality

The most likely cause of fetal death in this case is:


C. Placental abnormality: The presence of thrombi in the placental sam-
ple indicates that there was a problem with blood flow to the placenta,
which could have led to decreased oxygen and nutrient delivery to the
fetus. This can result in fetal growth restriction and, in severe cases, fetal
death.
While diabetes can increase the risk of complications during pregnan-
cy, including fetal death, the fact that this was an uneventful pregnancy
suggests that diabetes was not a significant contributing factor in this
case. Similarly, while fetal growth restriction can be a complication of
diabetes, the fetal weight of 2.6 kg is within the normal range and does
not suggest significant growth restriction.
Hypoglycemia would not be a likely cause of fetal death in this case, as it
typically does not occur without other signs of fetal distress.

Dr.Ali Balharith MD, MACP, FRCPC 58


Q62- Pregnant lady with active hepatitis b , What will you give tha
baby after birth ?
A. hepatitis b vaccine
B. hepatitis b vaccine and IG
C. IG

Correct answer is:B. hepatitis b vaccine and IG

In a case where a pregnant lady is diagnosed with active hepatitis B, it is


recommended to give both Hepatitis B vaccine and Hepatitis B immune
globulin (HBIG) to the baby after birth. Therefore, the correct option
would be B - hepatitis B vaccine and IG.
The reason for giving both HBV vaccine and HBIG to the newborn is to
provide passive and active immunity against the hepatitis B virus. The
passive immunity is provided by HBIG, which contains pre-formed an-
tibodies against the virus. The active immunity is provided by the hepa-
titis B vaccine, which stimulates the baby’s immune system to produce
their own antibodies against the virus.

Q63- A 40 year old women G4P4 , her last daughter age 8 And LMP
before 6 month , what u will check ?
A. FSH
B. TSH

Correct answer is:A. FSH

The most appropriate test to check would be follicle-stimulating


hormone (FSH) levels (Option A).

FSH is a hormone released by the pituitary gland that plays a crucial role
in regulating the menstrual cycle and promoting the growth and
development of ovarian follicles.

Dr.Ali Balharith MD, MACP, FRCPC 59


FSH levels are typically measured in women who are experiencing
infertility or irregular menstrual cycles as a part of the workup for
evaluating ovarian reserve and function.

In this case, the patient’s age and history suggest that she may be
experiencing perimenopause or menopause, and checking FSH levels
can help to confirm the diagnosis and assess her ovarian function.

TSH (Option B) is a hormone that is released by the pituitary gland and


plays a crucial role in regulating thyroid function.

It is typically measured in patients with suspected thyroid disease, such


as hypothyroidism or hyperthyroidism, rather than in patients with
suspected menopause or perimenopause.

Therefore, in a 40-year-old woman who is G4P4, and her last daughter


is 8 years old, and her LMP was 6 months ago, the most appropriate test
to check would be FSH levels to evaluate ovarian function and assess for
menopause or perimenopause.

Dr.Ali Balharith MD, MACP, FRCPC 60


Q64-A 40 year old women with Hx of DVT need to control her
birth ?
A. Condom
B. IUD
C. Patches
D. OCP

Correct answer is:A. Condom

In a 40-year-old woman with a history of DVT, the best option for birth
control would be option A - Condom.

Women with a history of DVT are at an increased risk of developing


another DVT, particularly when taking estrogen-containing
contraceptives such as oral contraceptive pills (OCP), hormonal patches,
or hormonal intrauterine devices (IUDs).

The use of these forms of birth control can increase the risk of blood
clots, which may lead to the development of DVT or even more severe
complications such as pulmonary embolism.

Condoms, on the other hand, are a non-hormonal form of birth control


that does not increase the risk of blood clots.

They are also effective in preventing the transmission of sexually trans-


mitted infections (STIs). Therefore, for women with a history of DVT,
condoms are the preferred and safest form of birth control.

Other non-hormonal options for birth control that could be considered


in this scenario include non-hormonal IUDs such as the copper IUD or
fertility awareness-based methods.
However, the use of these methods should be discussed with the pa-
tient’s healthcare provider, taking into account the patient’s individual
circumstances and preferences
Dr.Ali Balharith MD, MACP, FRCPC 61
Q65-A 44 years women is complaining of severe dysmenorrhea and
menorrhagia,pelvic examination reveals the uterus is symmetrical-
ly enlarged and tender.
Endometrial biopsy is normal Which of following dx?
A. Adenomyosis
B. Liomyomas
C. Endometriosis

Correct answer is:A. Adenomyosis

Adenomyosis is a condition where the endometrial tissue, which is nor-


mally located within the uterus, grows into the muscular wall of the uter-
us. This causes the uterus to become enlarged, tender, and symmetrical-
ly bulky. Adenomyosis can cause severe dysmenorrhea and menorrhagia,
which are symptoms reported by the patient in this case. Endometrial
biopsy is usually normal in adenomyosis.
Uterine leiomyomas (Option B), also called fibroids, are benign tumors
of the uterus that can cause pain and heavy bleeding. However, they
typically present as palpable masses on pelvic examination. They are not
usually associated with a symmetrically enlarged and tender uterus.
Endometriosis (Option C) is a condition where endometrial tissue grows
outside of the uterus, causing pain and menstrual irregularities. While
endometriosis can cause pelvic pain, it is not typically associated with a
symmetrically enlarged and tender uterus.
Therefore, based on the information provided, the most likely diagnosis
is Adenomyosis (Option A).

Dr.Ali Balharith MD, MACP, FRCPC 62


Q66- A Female patient with premenstrual pain what best to
decrease the pain?
A. Aerobic exercise
B. Decaf coffee

Correct answer is: A. Aerobic exercise is the best option to decrease


premenstrual pain.

Explanation:
- Premenstrual pain is a common symptom in women that occurs before
or during menstruation. Exercise has been shown to be effective in re-
ducing premenstrual pain.
- Aerobic exercise, in particular, can help release endorphins, which are
natural painkillers that can help relieve discomfort. Additionally, regular
exercise can help reduce stress and improve overall health.
- Decaf coffee does not have any direct effect on premenstrual pain and
therefore is not an effective treatment option.

Dr.Ali Balharith MD, MACP, FRCPC 63


Q67- A case scenario of open wound fracture who is stable and al-
ready received antibiotics, what do you want
to do next?
A. Closed reduction with above knee cast
B. Closed reduction with below knee cast
C. open reduction with intramedullary nail
D. Debridement with intramedullary nail

Correct answer is:D. Debridement with intramedullary nail

None of the options provided would be the most appropriate next step
for a stable patient with an open wound fracture who has already
received antibiotics.
The most appropriate next step in the management of an open wound
fracture in a stable patient who has already received antibiotics is
surgical debridement of the wound. This will involve removing any
debris or necrotic tissue from the wound and irrigating it thoroughly to
prevent infection and promote healing. This can be performed in the
operating room under local or general anesthesia.
Closed reduction with casting or open reduction with intramedullary
nailing are interventions that may be necessary in the management of
certain types of fractures, but they are not appropriate as the next step
for an open wound fracture in a stable patient who has already received
antibiotics.
Therefore, the most appropriate option for this case scenario would be
to perform surgical debridement of the wound.
D. Debridement with intramedullary nail is the appropriate next step.
Explanation:
- An open wound fracture is a serious injury that requires prompt treat-
ment to prevent infection and promote healing.
- The first step in the management of an open wound fracture is to
administer antibiotics to prevent infection.
- The next step is to perform a thorough debridement of the wound to
remove any foreign material or dead tissue that could harbor bacteria
Dr.Ali Balharith MD, MACP, FRCPC 64
and prevent healing.
- Intramedullary nailing is a surgical technique used to stabilize the bone
and promote healing after debridement.
- Closed reduction with above or below knee cast is not appropriate for
an open wound fracture, as it can increase the risk of infection and delay
healing.
- Therefore, debridement with intramedullary nail fixation is the most
appropriate next step in the management of an open wound fracture.

Dr.Ali Balharith MD, MACP, FRCPC 65


Q68- A Pregnant patient becomes diabetic during her pregnancy.
What is the best Management ?
A. Insulin
B. Metformin
C. DPP4 inhibitors

Correct answer is:A. Insulin

Explanation:

When a pregnant patient becomes diabetic during pregnancy, either as


gestational diabetes or pre-existing diabetes, the most commonly used
medication for management is insulin. Insulin is preferred because it
does not cross the placenta and is therefore safer for both the mother
and the fetus.
Metformin and dipeptidyl peptidase-4 (DPP4) inhibitors are oral med-
ications used in the management of type 2 diabetes, but they are not
recommended for use in pregnant women. The safety of these medi-
cations in pregnancy has not been established, and insulin remains the
most effective and safe medication for management of diabetes during
pregnancy.

Dr.Ali Balharith MD, MACP, FRCPC 66


Q69- White vaginal discharge but no rash, no odor. What is the or-
ganism/name of disease?
A. Bacterial vaginosis
B. trichomoniasis
C. candidiasis
D. UTI

Correct answer is: C. Candidiasis


Explanation:
- Candidiasis is a fungal infection caused by an overgrowth of Candida,
a type of yeast that is normally present in the vagina in small quantities.
- Symptoms of candidiasis include white, thick, and clumpy vaginal dis-
charge, as well as itching and irritation in the vulvar area.
- Unlike bacterial vaginosis and trichomoniasis, candidiasis does not typ-
ically have a strong odor associated with it.
- UTI, or urinary tract infection, is a different type of infection that af-
fects the urethra, bladder, or kidneys and is typically associated with
symptoms such as painful urination, frequent urination, and lower ab-
dominal pain. It is not typically associated with vaginal discharge.

Dr.Ali Balharith MD, MACP, FRCPC 67


Q70- A 70 years old female with prosthetic heart valve on warfarin,
has grade 3 pelvic organ prolapse. How to manage?
A. Pessary
B. Sacrocolpopexy
C. Le Fort Technique
D. Hystrectomy

Correct answer is:A. Pessary

Explanation:
In a 70-year-old female with a prosthetic heart valve who is on warfarin,
conservative management is recommended for pelvic organ prolapse
(POP). While surgical management may be an option, the risks of surgery
in this patient population are high due to the increased risk of bleeding
and other complications associated with anticoagulant therapy.
A pessary is a non-surgical option for the management of POP. It is a
small, removable device that is inserted into the vagina to support the
prolapsed pelvic organs. Pessaries are available in different shapes and
sizes and can be fitted to the patient’s individual needs.
Sacrocolpopexy is a surgical procedure that is used to correct POP by
attaching a surgical mesh to the sacrum and the anterior vaginal wall
to provide support to the pelvic organs. However, due to the risks asso-
ciated with anticoagulant therapy, surgery would not be the preferred
option in this case.
Le Fort technique and hysterectomy are surgical procedures that are not
typically used for the management of POP.
Therefore, pessary would be the safer and more appropriate manage-
ment option for this patient with a prosthetic heart valve on warfarin.

Dr.Ali Balharith MD, MACP, FRCPC 68


Q71-A 70 years old woman, sexually active, hysterectomy done
before, and she complains of prolapse and enlarged vaginal
opening, what is the appropriate procedure?
A. Sacrospinous fixation
B. Ant and post colpoperineorrhaphy
C. Manchester repair

Correct answer is: B. B. Ant and post colpoperineorrhaphy

Anterior and posterior colporrhaphy would be the appropriate


procedure to manage prolapse and enlarged vaginal opening in a 70-year-
old woman who is sexually active and has had a hysterectomy.

Explanation:
- Anterior and posterior colporrhaphy is a surgical procedure that in-
volves repairing and tightening the walls of the vagina to correct pro-
lapse and reduce the size of the vaginal opening.
- This procedure is appropriate for women who have had a hysterectomy
and are sexually active, as it does not involve the use of mesh or other
materials that could cause complications or discomfort during sexual ac-
tivity.
- Sacrospinous fixation is a surgical procedure used to correct prolapse of
the uterus, cervix, or vaginal vault, but it is not typically used to correct
prolapse of the vaginal walls.
- Manchester repair is a type of vaginal hysterectomy that involves the
removal of the uterus and the repair of the vaginal walls, and it is not
typically performed in women who have already had a hysterectomy.
- Therefore, anterior and posterior colporrhaphy is the most appropriate
procedure to manage prolapse and enlarged vaginal opening in a 70-year-
old woman who is sexually active and has had a hysterectomy.

Dr.Ali Balharith MD, MACP, FRCPC 69


Q72- Pregnant lady came for check up at 20 weeks gestation High
hba1c baby at risk of?
A. Congenital malformations
B. Chromosomal abnormality

Correct answer is: A. Congenital malformations are a potential risk


for a baby if the mother has high HbA1c levels at 20 weeks gestation.

Explanation:
- HbA1c is a test used to measure a person’s average blood sugar levels
over the past 2-3 months.
- High HbA1c levels in a pregnant woman indicate poor control of blood
sugar levels, which can have adverse effects on fetal development and
increase the risk of complications.
- Congenital malformations are a potential risk for a baby if the mother
has high HbA1c levels during pregnancy, particularly in the first trimes-
ter when the baby’s organs are developing.
- Chromosomal abnormalities, on the other hand, are typically not relat-
ed to the mother’s blood sugar levels during pregnancy.
- Therefore, if a pregnant woman has high HbA1c levels at 20 weeks gesta-
tion, it is important to closely monitor the pregnancy and manage blood
sugar levels to reduce the risk of congenital malformations and other
complications.

Dr.Ali Balharith MD, MACP, FRCPC 70


Q73- Pregnant lady has an epilepsy and smoke then she got prema-
ture labor asking about what was the big risk factor
A. Smoking
B. Epilepsy

Correct answer is: A. Smoking.

Smoking during pregnancy is a significant risk factor for premature labor


and delivery. Nicotine and other harmful chemicals in tobacco smoke
can cause constriction of blood vessels in the uterus, decrease blood flow
to the placenta, and lead to inflammation and oxidative stress, which can
initiate premature labor.
While epilepsy may be associated with certain obstetric risks, it is not
a direct cause of premature labor. However, pregnant women with epi-
lepsy may be at increased risk for certain complications, such as pre-ec-
lampsia, preterm labor, and fetal growth restriction, depending on the
severity of their epilepsy and the medications they are taking to manage
their condition. Therefore, both factors may contribute to the risk of
premature delivery, but smoking is a more significant risk factor in this
case.

Dr.Ali Balharith MD, MACP, FRCPC 71


Q74- Couple going to marriage, he have siblings with SCD , he is SC
trait, she is normal and have no family history , what is the risk if
they got married having children with SCD?
A. Very high
B. Low or nilll
C. They can’t get married

Correct answer is:B. Low or nilll

Explanation:
- Inheritance of SCD follows an autosomal recessive pattern, which
means that a child must inherit two copies of the abnormal hemoglobin
gene (one from each parent) to develop SCD.
- The man in this scenario has sickle cell trait (one normal hemoglobin
gene and one abnormal hemoglobin gene), which means he can pass ei-
ther the normal or abnormal gene to his offspring.
- The woman in this scenario has no family history of SCD and is pre-
sumed to have two normal hemoglobin genes.
- The Punnett square for this scenario would show a 50% chance of each
offspring inheriting the normal hemoglobin gene from the father and a
50% chance of inheriting the abnormal hemoglobin gene from the father.
- However, since the mother has two normal hemoglobin genes, even if
the offspring inherits the abnormal hemoglobin gene from the father,
they will still be a carrier of sickle cell trait (like the father) and will not
develop SCD.
- Therefore, the risk of having children with SCD in this case is low or nil.
However, genetic counseling may still be recommended for the couple
to discuss their options and potential risks.

Dr.Ali Balharith MD, MACP, FRCPC 72


Q75- Patient has heavy bleeding and usually has to change tampon
15-30 times/day, recently she has been engaged in intercourse with
multiple partners to conceive, on examination: retroverted uterus,
decreased uterine mobility , and tender uterosacral ligament with
nodularity. What’s the diagnosis?
A. Endometriosis.
B. PID
C. Fibroid
D. Adenomyosis

Correct answer is: A. Endometriosis.

The patient’s symptoms of heavy bleeding, need to change tampons fre-


quently, painful intercourse, and tender uterosacral ligament with nodu-
larity suggest a diagnosis of endometriosis. The presence of a retroverted
uterus and decreased uterine mobility are also commonly seen in cases
of endometriosis.
Endometriosis occurs when tissue similar to the lining of the uterus
grows outside of the uterus, and it can cause pelvic pain, painful bowel
movements or urination, infertility, and other symptoms. Endometrio-
sis can also lead to adhesions and scarring, which may cause decreased
mobility of the uterus and uterosacral ligament tenderness.
PID (Pelvic Inflammatory Disease) is an infection of the reproductive
organs that can also cause pelvic pain, heavy bleeding and painful inter-
course. However, it is less likely to cause uterosacral ligament tender-
ness with nodularity.
Fibroids are noncancerous growths that can cause heavy menstrual
bleeding, but they are not typically associated with tender uterosacral
ligament nodularity or retroverted uterus.
Adenomyosis is a condition in which the inner lining of the uterus (en-
dometrium) grows into the muscular wall of the uterus, causing heavy or
prolonged menstrual bleeding, painful periods, and an enlarged uterus.
However, it is less likely to cause uterosacral ligament tenderness with
nodularity and retroverted uterus.
Dr.Ali Balharith MD, MACP, FRCPC 73
Q76- Endometriosis . took nsaids no improvement what is the treat-
ment?
A. Laparoscopy
B. OCP

Correct answer is: A. Laparoscopy.

If taking NSAIDs does not improve symptoms of endometriosis, laparos-


copy is the next step in the management of endometriosis. Laparoscopy
is the gold standard diagnostic and therapeutic tool for endometriosis as
it allows direct visualization and excision of endometrial tissue.
Oral contraceptives (OCPs) are also commonly used in the treatment of
endometriosis, as they can reduce menstrual pain and bleeding by sup-
pressing ovulation and thinning the lining of the uterus. However, they
are not the first-line treatment if NSAIDs are not effective, and they are
often used as a long-term therapy for endometriosis management.
Other treatment options for endometriosis include progestins, andro-
genic agents such as danazol or gestrinone, gonadotropin-releasing hor-
mone (GnRH) agonists, and aromatase inhibitors. The choice of treat-
ment will depend on various factors such as the severity of symptoms,
age, desire for pregnancy, and patient preference and should be deter-
mined by a healthcare professional.

Dr.Ali Balharith MD, MACP, FRCPC 74


Q77- Female prenatal visit history of hypothyroidism on Levothy-
roxine , it’s controlled and T4 tsh level are normal
What to do?
A. Increase dose
B. Continue same dose
C. Decrease dose

Correct answer is: A. Increase dose

it may be necessary to increase the dose of levothyroxine in pregnant


women with hypothyroidism, even if their T4 and TSH levels are cur-
rently within the normal range.
Explanation:
- During pregnancy, the demand for thyroid hormone increases, and some
women with hypothyroidism may require an increase in their levothy-
roxine dose to maintain adequate thyroid hormone levels.
- The American Thyroid Association recommends that pregnant women
with hypothyroidism should have their levothyroxine dose adjusted to
maintain a TSH level in the lower reference range (ideally <2.5 mIU/L) in
the first trimester, and to maintain TSH levels between the lower refer-
ence range and 3.0 mIU/L in the second and third trimesters.
- Therefore, in this scenario, if the patient’s TSH level is at the upper end
of the normal range or above 2.5 mIU/L in the first trimester, it may be
appropriate to increase her levothyroxine dose to maintain adequate thy-
roid hormone levels and prevent adverse outcomes such as miscarriage,
preterm delivery, and impaired neurodevelopment in the offspring.
- It is important for pregnant women with hypothyroidism to work close-
ly with their healthcare provider to monitor their thyroid function and
adjust their levothyroxine dose as needed to maintain optimal thyroid
hormone levels during pregnancy.

Dr.Ali Balharith MD, MACP, FRCPC 75


Q78- A 23 wife medically free, regular cycle, c/o primary infertility
for 3 months, Husband 26, medically free, has 1 cousin autustic, 1
with trisomy 21, Most appropriate :
A. Continue trying
B. Semen count
C. Antenatal testing

Correct answer is: A. Continue trying.

In this scenario, the most appropriate step would be to continue trying


for pregnancy for a little while longer. The couple has only been trying to
conceive for 3 months, which is a relatively short time for trying to get
pregnant.
Primary infertility is defined as not getting pregnant after trying for at
least 12 consecutive months, which means this couple has not yet reached
the definition of infertility.
While the presence of a family history of autism and trisomy 21 may
increase the risk of these conditions in the offspring, it is not a reason
to initiate genetic testing or medical intervention for infertility at this
point.
Additionally, the couple’s ages and medical history are generally favor-
able for fertility, and the wife reports a regular menstrual cycle, which
suggests potential regular ovulation.

B. Semen count and C. Antenatal testing are not appropriate next steps
in the management of primary infertility in this case, and therefore they
are considered incorrect options.
B. Semen count is a test that assesses the number and quality of sperm
in a semen sample. However, since the wife reports a regular menstrual
cycle and the couple has only been trying to conceive for 3 months, it
is not clear whether male-factor infertility is a contributing factor, and
Semen analysis does not guarantee fertility.
C. Antenatal testing aims to screen or diagnose certain conditions or ab-
normalities that may be present in the fetus or the mother during preg-
Dr.Ali Balharith MD, MACP, FRCPC 76
nancy. However, antenatal testing is not a diagnostic tool for infertility,
and it is not typically used in cases of primary infertility.

Q79- Pregnant 31 weeks came with uterine contraction and cervix


closed what to give
A. Nifedipine
B. Terbutaline
C. Indomethacin
D. Corticoseroid

Correct answer is: A. A. Nifedipine

Nifedipine is the most appropriate medication to give to a pregnant


woman at 31 weeks gestation who presents with uterine contractions
and a closed cervix.

Explanation:
- Nifedipine is a calcium channel blocker that can be used to relax smooth
muscle, including the smooth muscle of the uterus.
- It is often used to manage preterm labor by reducing the frequency and
intensity of uterine contractions.
- Terbutaline is another medication that can be used to relax the smooth
muscle of the uterus, but it is associated with a higher risk of adverse ef-
fects, such as tachycardia and hypotension, and is typically reserved for
cases in which other medications have failed.
- Indomethacin is a nonsteroidal anti-inflammatory drug (NSAID) that
can also be used to manage preterm labor, but it is associated with a
higher risk of adverse effects, such as premature closure of the ductus
arteriosus in the fetus, and is typically reserved for cases in which other
medications have failed.
- Corticosteroids may be given to enhance fetal lung maturity in cases
where preterm delivery is imminent, but they are not typically used to
manage uterine contractions.
Dr.Ali Balharith MD, MACP, FRCPC 77
- Therefore, in this scenario, nifedipine is the most appropriate
medication to give to manage uterine contractions and reduce the risk
of preterm delivery.

Q80- A young female on labor GA 38/40. Her pregnancy was normal,


uneventful, with normal fetal development. The labor was normal
but the was difficulty delivering the placenta which complicated
with uterine inversion accompanied with vaginal bleeding 1200cc.
Which of the following is the source of her bleeding?
A. Trauma blood vessels
B. Muscular injury and laceration
C. Uterine muscles fail to contract

Correct answer is:C. The uterine muscles fail to contract is the like-
ly source of her bleeding in this scenario.

Explanation:
- Uterine inversion is a rare but serious complication of childbirth in
which the uterus turns inside out after the delivery of the baby and pla-
centa.
- Uterine inversion can be life-threatening and can cause heavy vaginal
bleeding and shock.
- One of the main causes of bleeding in uterine inversion is the failure of
the uterine muscles to contract properly.
- This can be due to a variety of factors, including over-distension of the
uterus, rapid delivery of the placenta, or excessive traction on the umbil-
ical cord.
- Trauma to blood vessels or muscular injury and laceration can also cause
bleeding during childbirth, but in this scenario, the bleeding is likely due
to the failure of the uterine muscles to contract properly, which is a com-
mon cause of bleeding in cases of uterine inversion.
- It is important to quickly manage uterine inversion and control bleed-
ing to prevent further complications and ensure the patient’s safety.

Dr.Ali Balharith MD, MACP, FRCPC 78


Q81- Pregnant lady with Leg swelling and sign of DVT , SOB , what
best next ?
A. CT PA
B. Chest x-ray
C. Doppler US of LLL
D. V/Q scan

Correct answer is: C. Doppler US of LLL

Explanation: Leg swelling and signs of deep vein thrombosis (DVT) along
with shortness of breath (SOB) is concerning for a possible pulmonary
embolism (PE). PE commonly arises from deep vein thrombosis (DVT)
in the legs. Thus, Doppler ultrasound of the lower limb (LLL) is a good
initial test to confirm the presence of DVT in this pregnant lady. CT PA
scan can be considered as the next step if Doppler ultrasound is incon-
clusive or technically difficult. Chest x-ray and V/Q scan are not helpful
in diagnosis of DVT.

Q80- A 26-year-old female came to the Clinic complaining of severe


lower abdominal pain tat started just before her menses and disap-
peared on the 3rd day, the pain so severe that affect her daily activ-
ity and prevent her from going to work for many days. Her boss is
not happy. Which of the following is the most appropriate manage-
ment?
A. hysterectomy
B. Paracetamol
C. OCP

Correct answer is: C. OCP (Oral Contraceptive Pills)

Explanation: The 26-year-old female who is experiencing severe cyclic


lower abdominal pain that starts just before her menses and disappears
on the 3rd day is highly suggestive of endometriosis. Endometriosis is a
gynecologic condition where the tissue that usually lines the inside of
Dr.Ali Balharith MD, MACP, FRCPC 79
the uterus grows outside of it. It causes pain and discomfort, especially
during menstruation.
The recommended treatment options for endometriosis are medical or
surgical management as per the severity of the symptoms. As this is a
mild to a moderate case, the most appropriate treatment choice would
be OCPs. OCPs help reduce the growth of endometrial tissue and hence
help in alleviating pain symptoms caused by endometriosis. Surgical op-
tions like hysterectomy should be considered only in severe cases that
are unresponsive to medical treatments. Paracetamol may provide symp-
tomatic relief but is not the appropriate treatment for endometriosis.
It is important to educate the patient about the nature of endometriosis
and the management plan, along with its benefits and side effects, and
the need for follow-up to ensure efficacy and adjust the treatment if nec-
essary.

Q82- What of the following will cause oligohydramnios?


A. Placental insufficiency
B. DM
C. Chriongioma
D. Dudenal atresia

Correct answer is: A. Placental insufficiency is one of the causes of


oligohydramnios.

Explanation:
- Oligohydramnios is a condition in which there is a decreased amount of
amniotic fluid surrounding the fetus.
- Amniotic fluid is important for fetal growth and development, and oli-
gohydramnios can lead to complications such as fetal growth restriction,
preterm labor, and fetal distress.
- Placental insufficiency is a condition in which the placenta is unable to
supply the fetus with adequate nutrients and oxygen.
- Placental insufficiency can lead to oligohydramnios because the de-
creased blood flow to the placenta can cause a decrease in fetal urine
Dr.Ali Balharith MD, MACP, FRCPC 80
output and a subsequent decrease in the amount of amniotic fluid.
- Other causes of oligohydramnios include ruptured membranes, fetal
renal abnormalities, and maternal hypertension.
- DM (diabetes mellitus), choriangioma, and duodenal atresia are not typ-
ically associated with oligohydramnios.

Q83- What considered the Best score on biophysical profile?


A. 8/10
B. 9/10
C. 12/1

Correct answer is: B. 9/10

Explanation: The biophysical profile (BPP) is a prenatal test that evaluates


fetal well-being by measuring five parameters, including fetal breathing
movements, fetal body movements, fetal muscle tone, amniotic fluid
volume, and fetal heart rate. Each parameter is given a score of 0 or 2,
and the scores are added up to give the overall BPP score.

The highest possible score on the BPP is 10/10, which indicates a healthy,
active fetus with a sufficient amniotic fluid volume. However, some scor-
ing systems allow for a maximum score of 8/10, or even 6/10, depending
on the specifics of the scoring system. A BPP score of 9/10 is considered
the best score as per the usual scoring system as it indicates that the fe-
tus is doing well and there are no significant risks of fetal compromise or
distress. A score of 8/10 or below may indicate the need for closer mon-
itoring or more extensive testing, depending on the clinical situation. A
score of 12/1 is not a relevant or appropriate score in the context of the
BPP.

Dr.Ali Balharith MD, MACP, FRCPC 81


Q84- A case scenario of patient came to clinic with preeclampsia
without severe features at 32 weeks . What you do?
A. Admission for observation
B. Give appointment at outpatient clinic after 1 week

Correct answer is: A. Admission for observation

Explanation: Preeclampsia is a pregnancy-related complication charac-


terized by high blood pressure and damage to organs such as the liver
and kidneys. In the absence of severe features at 32 weeks, the standard
of care is to admit the patient for close observation since preeclampsia
can progress rapidly and develop into a severe condition such as eclamp-
sia (seizures associated with preeclampsia) or HELLP (hemolysis, elevat-
ed liver enzymes, and low platelets) syndrome.

Admission to the hospital allows for careful monitoring of the patient’s


blood pressure, urine output, and laboratory values. The fetus can also be
closely observed for signs of distress or compromised fetal well-being.
The management of preeclampsia is aimed at preventing the progres-
sion of the disease and the development of complications. It may involve
the use of medications to lower blood pressure or the induction of labor
if the condition worsens or the fetus is at risk.

Giving an outpatient clinic appointment after one week is not appro-


priate in this situation since preeclampsia can progress rapidly and may
require urgent intervention. Prompt treatment and close follow-up are
required to ensure the best possible outcomes for both the mother and
the baby.

Dr.Ali Balharith MD, MACP, FRCPC 82


Q85- A Pregnant with hx of previous DVT , what to do ?
A. Warfarin
B. LMWH
C. No need for anticoagulation

Correct answer is: B. Low molecular weight heparin (LMWH)

Explanation:
- Pregnant women are at an increased risk of developing DVT due to
changes in blood clotting factors and venous stasis.
- Women with a history of previous DVT are at an even higher risk and
require anticoagulation therapy to prevent recurrence.
- Warfarin is contraindicated during pregnancy due to its teratogenic ef-
fects, as it can cross the placenta and cause fetal abnormalities.
- LMWH is the preferred anticoagulation therapy for pregnant women
with a history of DVT, as it does not cross the placenta and has a low risk
of fetal complications.
- LMWH is administered subcutaneously and does not require monitor-
ing of coagulation parameters.
- LMWH can be safely used throughout pregnancy and is typically con-
tinued for 6 weeks after delivery to reduce the risk of postpartum DVT.
- Therefore, in this scenario, LMWH is the recommended anticoagula-
tion therapy for a pregnant woman with a history of previous DVT.

Dr.Ali Balharith MD, MACP, FRCPC 83


Q86- A Primigravida come with labor for 4h Dilated 5 cm, effaced
80%, station +1 after 5h there is no change in cervix, and contraction
occur every 3 min. and stay for 60 Sec. What to do?
A. Instrument use
B.C/S
C. IV oxytocin
D. Wait for 2h

Correct answer is: C. IV oxytocin

Explanation: Prolonged labor or failure to progress in labor is a common


complication seen in pregnancy, especially in primigravida women. The
diagnosis is established when there is no cervical change for 4 hours in
active labor. If there is no change in the cervix for an additional 2 hours,
it is highly suggestive of failure to progress in labor.

In this scenario, since there has been no change in cervical dilatation or


effacement after 5 hours, the next step would be to augment labor by
using oxytocin. Oxytocin is a hormone that stimulates uterine contrac-
tions and can help progress labor. It can be administered intravenously
to increase the frequency and intensity of contractions leading to cervi-
cal dilatation. Careful monitoring of mother and fetus is essential when
oxytocin is used as excessive uterine contractions can lead to fetal dis-
tress.

Instrumental delivery or cesarean section should be avoided in a case of


prolonged labor unless there is an emergency indication. While waiting
for two more hours is an option; prolonging the labor process may in-
crease the risk of maternal and neonatal complications, making it not a
practical option in this case.

Therefore, the most appropriate management approach for this patient


is to initiate IV oxytocin to augment labor with adequate fetal and mater-
nal monitoring, aiming for vaginal delivery.
Dr.Ali Balharith MD, MACP, FRCPC 84
Q87- A 34 year old Female primigravida, smoker, came for antena-
tal visit, history of subfertility, and family history of diabetes mili-
tus in second degree relative. What’s the biggest risk factor for her?
A. Smoking
B. Subfertility
C. Age
D. Family history of DM

Correct answer is: A. Smoking

Explanation:
Among the given risk factors of smoking, subfertility, advanced mater-
nal age, and family history of diabetes mellitus in a second-degree rela-
tive, the biggest risk factor for the 34-year-old primigravida is smoking.
Smoking during pregnancy is a significant risk factor for adverse preg-
nancy outcomes, including preterm labor, growth restriction, placental
abruption, stillbirth, and congenital anomalies. Smoking also increases
the risk of maternal complications such as pre-eclampsia, placenta pre-
via, and premature rupture of the membranes.
Subfertility, advanced maternal age, and family history of diabetes melli-
tus in second-degree relatives are also significant risk factors for adverse
pregnancy outcomes, but their impact is generally less severe than smok-
ing. Women with subfertility usually receive medical assistance, and ade-
quate preconception counseling and care should help reduce the risk of
adverse pregnancy outcomes. Advanced maternal age is associated with
increased fetal aneuploidy and increased obstetric complications. Family
history of diabetes mellitus in a second-degree relative increases ante-
natal screening, and adequate glycemic control during pregnancy would
minimize adverse outcomes.
However, since all the mentioned factors can contribute to an increased
risk of adverse pregnancy outcomes, preconception counseling, risk as-
sessment, and individualized care planning are essential to achieve bet-
ter maternal and fetal outcomes. Antenatal care plays a vital role in re-
ducing the impact of risk factors and providing better prenatal care for
Dr.Ali Balharith MD, MACP, FRCPC 85
achieving better fetal outcomes.

Q88- Highest causing factor of endometrial cancer:


A. late menarche.
B. Early menopause.
C. DM.
D. progesterone tumor

Correct answer is: D. Progesterone tumor

Explanation: The highest causing factor of endometrial cancer is related


to estrogen exposure without an opposing effect of progesterone.
Estrogen is a hormone that stimulates the growth and proliferation of the
endometrial tissue, while progesterone plays a counterbalancing role by
inducing differentiation and maintaining the integrity of the endometri-
al tissue by regulating the effects of estrogen. When there is unopposed
estrogen exposure, leading to excessive growth of the endometrial tissue
without adequate differentiation or shedding, it can eventually result in
endometrial hyperplasia, a precursor to endometrial cancer.
A progesterone tumor occurs when there is a deficiency of progester-
one relative to estrogen, leading to unopposed estrogen exposure, and
promoting endometrial hyperplasia and cancer. Endometrial cancer is a
hormone-dependent cancer, and the use of estrogen replacement thera-
py without progesterone is a known risk factor.
Late age at menarche, early age at menopause, and diabetes mellitus are
other risk factors for endometrial cancer, but they are not the highest
causing factor. Age at menarche and menopause are related to cumu-
lative exposure to estrogen. Early age at menopause results in a short-
er duration of endometrial exposure to estrogen. Diabetes Mellitus, a
chronic metabolic disease, is related to an increased risk of cancer due to
increased insulin resistance, which results in an increased level of insulin
and IGF-1 levels, leading to proliferation of endometrial tissues.

Dr.Ali Balharith MD, MACP, FRCPC 86


Q89- A 30 years old female with right Ovarian cyst . asking what is
the best contraception for her ?
A. Progestin only
B. OCP
C. Condom

Correct answer is: A. Progestin only also known as the mini-pill.

Progestin-only contraception works by thickening the cervical mucus,


which makes it difficult for sperm to reach the egg. It also thins the lin-
ing of the uterus, which can make it difficult for a fertilized egg to im-
plant. It is generally considered safe for women with ovarian cysts and
has fewer side effects than estrogen-containing contraceptives.
Combined oral contraceptive pills (OCPs) contain both estrogen and
progesterone, which can potentially stimulate the growth of the ovarian
cyst.
Barrier methods like condoms are not effective enough as a sole contra-
ception method.
Therefore, progestin-only methods, such as the mini-pill, the implant, or
the hormonal IUD, are better options as they only contain progesterone
and do not increase the risk of cyst growth.

Dr.Ali Balharith MD, MACP, FRCPC 87


Q90-After endometrial biopsy in a 27 year old, ATYPICAL HYPER-
PLASIA SEEN, what is the next management ?
A. Tamoxifen
B. Progesterone-only
C. OCP
D. Hysterectomy

Correct answer is: D. Hysterectomy

If atypical hyperplasia is detected on endometrial biopsy in a 27-year-old


woman, the next management step would be Option D, Hysterectomy.
Atypical hyperplasia is a pre-cancerous condition of the endometrium,
which carries an increased risk of developing endometrial cancer. It is
often associated with abnormal uterine bleeding and can be diagnosed
by endometrial biopsy.
While options such as tamoxifen (Option A), progesterone-only therapy
(Option B), and oral contraceptive pills (Option C) may be considered
in certain cases, they are not the first-line management for atypical hy-
perplasia. In young women, the conservative approach is often desired,
however, in this case, given the severity of the pathology and the poten-
tial risk of progression to endometrial cancer, a hysterectomy would be
recommended as it is the definitive treatment option for atypical hyper-
plasia.

Dr.Ali Balharith MD, MACP, FRCPC 88


Q91- A 23 year old regnant women came to antenatal clinic with hx
of 2 preterm birth C/O vaginal spotting of blood , which of the fol-
lowing recommended tx ?
A. Progesterone
B. Estrogen
C. Indomethacin
D. MgSo4

Correct answer is: A. Progesterone.

Progesterone supplementation is recommended for women with a his-


tory of preterm birth in order to reduce the risk of recurrent preterm
birth. Vaginal spotting can be a sign of cervical shortening, which is a risk
factor for preterm birth. Progesterone supplementation has been shown
to reduce the risk of preterm birth in women with a short cervix.
B. Estrogen is not recommended as a treatment for vaginal spotting in
pregnant women and has no role in the prevention of preterm birth.
C. Indomethacin is a nonsteroidal anti-inflammatory drug (NSAID) that
is sometimes used to stop preterm labor, but it is not indicated for vagi-
nal spotting and may have negative effects on fetal development.
D. MgSo4 (magnesium sulfate) is used for fetal neuroprotection in certain
high-risk pregnancies, such as those with preterm labor or preeclampsia,
but it is not indicated for vaginal spotting or prevention of preterm birth
in women with a history of preterm birth.

Dr.Ali Balharith MD, MACP, FRCPC 89


Q92- 39 years old female who has three children and completed her
family diagnosed as endometrioma which was removed 2 years ago,
right ovary cyst she presented to the clinic with mild to moderate
dysmenorrhea and dyspareunia during intercourse and chronic
lower abdominal pain. Pelvic ultrasound shows: Left ovary endo-
metrioma cyst 6x7 in size.
A. Removal of cyst more than 10 in size
B. Aspiration of cyst content under ultrasound guidance
C. Immediate hysterectomy and salpingectomy oophorectomy
D. Removal of cyst by laparoscopic ablation of endometrioma spots.

Correct answer is: D. Removal of cyst by laparoscopic ablation of


endometrioma spots.

In cases of endometriomas, the most appropriate surgical treatment is


typically laparoscopic excision of the cyst wall with concurrent remov-
al of endometriosis implants. This approach aims to preserve as much
ovarian tissue as possible while removing the disease completely.
A. Removal of a cyst larger than 10 cm is typically not performed laparo-
scopically and may require an open laparotomy, which carries a higher
risk of complications than a laparoscopic approach. In addition, operative
intervention on small asymptomatic endometriomas is controversial.
B. Aspiration of the cyst content does not remove the cyst’s wall, which
is the source of pain and potential complications.
Plus, the recurrence rate of an endometrioma after aspiration is high.
C. Hysterectomy and oophorectomy are not appropriate treatments for
symptoms related to an endometrioma cyst. Removing both ovaries
could lead to premature menopause and potential long-term adverse
health outcomes.
Therefore, laparoscopic ablation of endometrioma spots would be the
most appropriate surgical treatment option for this patient.

Dr.Ali Balharith MD, MACP, FRCPC 90


Q93- 46 Y/O G3P1 at 34 weeks’ gestation presented to antenatal clin-
ic for regular check-up, she has unremarkable medical hx and un-
complicated pregnancy Braxton Hicks and non-pruritic cervical
discharge. Her pre-pregnancy weight was 54.4 on examination
cervical length was 33 mm.
VS were given & I believe they were normal.
Current weight: 52
Rubella AB: +ve
HBsAg: -ve
Blood type: O+
Which of the following is the most appropriate next step?
A. F/U after 2 weeks
B. OGGT test
C. Do rubella Ab test / Repeat rubella screen
D. Give anti-D Ab

Correct answer is: A. F/U after 2 weeks.

The patient is at 34 weeks of pregnancy, and her prenatal evaluation


does not reveal any concerning findings. She has a normal cervical length
of 33mm, and her vital signs are normal. Additionally, she has a positive
Rubella antibody and negative HBsAg.
A. Follow-up after 2 weeks would be the most appropriate next step be-
cause the patient does not have any significant findings that would re-
quire immediate management. This follow-up visit would allow for con-
tinued monitoring of cervical length and fetal growth.
B. An OGGT test is not indicated in this scenario since the patient doesn’t
have any concerning symptoms or risk factors for gestational diabetes.
C. Doing a repeat rubella screening or Rubella Ab test is not necessary in
this situation since the patient has a positive Rubella antibody, indicating
that she is immune.
D. Giving Anti-D Ab is not indicated at this time since the patient’s blood
is Rh(D) positive, and she does not have any concerning clinical findings
that would require such intervention.
Dr.Ali Balharith MD, MACP, FRCPC 91
Therefore, the most appropriate next step for this patient is to have a
follow-up visit after 2 weeks for continued monitoring of her pregnancy.

Q94- Patient presenting with yellowish to green vaginal discharge,


strawberry cervix, what is the treatment ?
A. Miconazold
B. Flucanzole
C. Metronidazole

Correct answer is: C. Metronidazole.

Yellowish to green vaginal discharge and strawberry cervix are symptoms


of bacterial vaginosis (BV), which is a common vaginal infection caused
by an overgrowth of anaerobic bacteria in the vagina. Metronidazole is
the first-line treatment for BV. It is an antibiotic that works by killing the
bacteria causing the infection.
Option A - Miconazole is an antifungal medication used to treat fungal
infections but would not be effective in treating BV.
Option B - Fluconazole is also an antifungal medication used to treat fun-
gal infections, but it would not be effective in treating BV.
Therefore, the correct answer is C. Metronidazole.

Dr.Ali Balharith MD, MACP, FRCPC 92


Q95- A 40y/o female patient underwent PAP smear histopathology
showed ASCUS, your next step?
A. Do HPV test
B. Colposcopy.
C. Re-evaluate after 6 months
D. Surgery

Correct answer is: B. Colposcopy.

ASCUS (Atypical Squamous Cells of Undetermined Significance) is a


finding on Pap smear that suggests the presence of abnormal squamous
cells in the cervix, but it is not diagnostic of a specific condition. The
next step in the evaluation of ASCUS is a colposcopy, a procedure where
the cervix is examined using a special microscope to look for signs of
cervical dysplasia or pre-cancerous changes.

Option A - Doing an HPV test may be useful in the evaluation of ASCUS,


but it is not the next step in the evaluation. Colposcopy is the next step
after ASCUS diagnosis.
Option C - Re-evaluating after 6 months without doing colposcopy in a
patient with ASCUS may delay the diagnosis of cervical abnormalities,
and hence is not recommended.
Option D - Surgery is not the next step in the evaluation of ASCUS. In
fact, surgery is not usually done for pre-cancerous changes in the cervix.
Treatment for pre-cancerous cervical abnormalities often involves cryo-
therapy, loop electrosurgical excision procedure (LEEP), or cone biopsy
based on the severity of the changes and the age of the patient.
Therefore, the correct answer is B. Colposcopy.

Dr.Ali Balharith MD, MACP, FRCPC 93


Q96- 3rd trimester UTI?
A. Nitrofurantoin
B. Trimethoprim-sulfamethoxazole

Correct answer is:A. Nitrofurantoin

Nitrofurantoin is a commonly used antibiotic for the treatment of UTIs


in pregnancy, particularly in the second and third trimesters. It has a low
risk of teratogenicity and is considered safe for both the mother and the
fetus. However, it should not be used in patients with renal impairment
or in the last few weeks of pregnancy due to the risk of hemolytic ane-
mia in newborns.
Trimethoprim-sulfamethoxazole (option B) is also effective for the treat-
ment of UTIs, but it is associated with a higher risk of birth defects, par-
ticularly in the first trimester. It should be avoided in the first trimester
and used with caution in the second and third trimesters.
Other antibiotics that may be considered for the treatment of UTIs in
pregnancy include amoxicillin, cephalosporins, and penicillins. Howev-
er, the choice of antibiotic should be based on the patient’s individu-
al characteristics, including allergies, renal function, and other medical
conditions.
Therefore, in a patient with a UTI in the third trimester of pregnancy,
the most appropriate treatment option would be A) Nitrofurantoin.

Dr.Ali Balharith MD, MACP, FRCPC 94


Q97- 26 female came with AUB Endometrial biopsy show atypical
cell Most appropriate management?
A. Hysterectomy
B. Tamoxifen

Correct answer is:A. Hysterectomy

Atypical cells on an endometrial biopsy are indicative of endometrial hy-


perplasia or endometrial cancer. In young patients, endometrial hyper-
plasia is more common than cancer and may progress to cancer if left
untreated. Therefore, it is important to establish the diagnosis and initi-
ate appropriate treatment.
Hysterectomy is the most definitive treatment for endometrial hyper-
plasia or cancer. It involves the removal of the uterus and cervix and is
curative in most cases. Hysterectomy may be performed as a total hys-
terectomy (removal of the uterus and cervix) or a radical hysterectomy
(removal of the uterus, cervix, and upper part of the vagina).
Tamoxifen (option B) is a medication used for the treatment of breast
cancer and is not appropriate for the treatment of endometrial hyperpla-
sia or cancer. In fact, tamoxifen use is a risk factor for the development
of endometrial hyperplasia and cancer and may exacerbate pre-existing
disease.
Therefore, in a 26-year-old female with AUB and an endometrial biopsy
showing atypical cells, the most appropriate management would be A)
Hysterectomy.

Dr.Ali Balharith MD, MACP, FRCPC 95


Q98- Pregnant G2p1 with history of SCA came for followup
previous pregnancy was vaginal delivery uneventful and now in 9
week asymptomatic she has previous 2 admission vasooclussive at-
tack HG 90 normal more than 120 Management ?
A. Simple transfusion
B. Exchange transfusion
C. Close followup
D. Hydroxyurea

Correct answer is:C. Close followup

In a pregnant patient with sickle cell anemia (SCA), who is G2P1 and in
the 9th week of pregnancy, with a history of two previous admissions
for vaso-occlusive crises (VOC) but asymptomatic at present, the most
appropriate management would be C) Close follow-up.

Sickle cell anemia is associated with an increased risk of complications


during pregnancy, which can include vaso-occlusive crises, acute chest
syndrome, pre-eclampsia, and fetal growth restriction. The patient’s pre-
vious history of VOC may increase the risk of complications during this
pregnancy.
Simple transfusion (option A) or exchange transfusion (option B) may be
considered in pregnant patients with SCA who develop severe anemia or
acute complications such as acute chest syndrome or stroke. However,
in the absence of symptoms or complications, routine transfusions are
not recommended during pregnancy.
Hydroxyurea (option D) is a medication that can reduce the frequency
and severity of VOC in patients with SCA. It can be used during pregnan-
cy in selected cases, but it requires close monitoring of the patient and
the fetus.
Therefore, in a pregnant patient with sickle cell anemia who is G2P1 and
in the 9th week of pregnancy, with a history of two previous admissions
for vaso-occlusive crises but asymptomatic at present, the most appro-
priate management would be C) Close follow-up. The patient should re-
Dr.Ali Balharith MD, MACP, FRCPC 96
ceive close monitoring for the development of complications, and ap-
propriate management should be initiated if any complications arise.

Q99- Asymptomatic, 5-6 cm fibroid. What is the management?


A. Follow up with Pelvic ultrasound yearly
B. Follow up with pelvic ultrasound and complete blood count ev-
ery two months

Correct answer is:A. Follow up with Pelvic ultrasound yearly

Fibroids are noncancerous growths in the uterus that are quite common,
especially in women of reproductive age. Many women have fibroids
that do not cause any symptoms or problems. In asymptomatic patients,
conservative management with regular monitoring is the preferred ap-
proach.
Pelvic ultrasound is a non-invasive imaging test that can be used to mon-
itor the size and growth of the fibroid. A yearly pelvic ultrasound (option
A) is appropriate for an asymptomatic patient with a 5-6 cm fibroid, as
this would allow for monitoring of any changes in size or appearance of
the fibroid.
A complete blood count (CBC) is not necessary for routine monitoring of
a fibroid and would not provide useful information in an asymptomatic
patient. Therefore, option B is not the most appropriate management for
an asymptomatic patient with a 5-6 cm fibroid.
In summary, in an asymptomatic patient with a 5-6 cm fibroid, the most
appropriate management would be A) Follow up with pelvic ultrasound
yearly. This allows for regular monitoring of the fibroid without subject-
ing the patient to unnecessary tests or interventions.

Dr.Ali Balharith MD, MACP, FRCPC 97


Q100- 45 year old woman, asymptomatic, have many kids and the
latest is a healthy 8 year old boy. Currently wants to get pregnant.
She has amenorrhea for 5 months. Pregnancy test negative. Which
of the following is the most appropriate investigation?
A. FSH
B. TSH

Correct answer is:A. FSH

Amenorrhea refers to the absence of menstrual periods, and it can be


caused by a variety of factors, including pregnancy, menopause, and cer-
tain medical conditions. In this case, since the pregnancy test is negative
and the patient wants to get pregnant, other causes of amenorrhea need
to be investigated.
FSH (follicle-stimulating hormone) is a hormone produced by the pitu-
itary gland that stimulates the growth and development of ovarian folli-
cles in women. High levels of FSH are typically indicative of ovarian fail-
ure or menopause, while low levels of FSH can indicate a hypothalamic
or pituitary problem.
In this scenario, since the patient is 45 years old and has amenorrhea,
testing the FSH levels (option A) can help determine if she is entering
menopause or has ovarian failure. If the FSH levels are high, this would
suggest that she is entering menopause, and the chances of getting preg-
nant naturally are very low. If the FSH levels are normal or low, further
investigations to find the underlying cause of amenorrhea can be done.
TSH (thyroid-stimulating hormone) testing (option B) may be useful if
there are other symptoms of thyroid dysfunction, such as weight gain,
fatigue, or hair loss, but it is less likely to be the cause of amenorrhea in
an otherwise asymptomatic patient.
Therefore, in a 45-year-old woman who is asymptomatic, has many chil-
dren including a healthy 8-year-old boy, and has amenorrhea for 5 months
with a negative pregnancy test, the most appropriate investigation is A)
FSH.

Dr.Ali Balharith MD, MACP, FRCPC 98


Q101- Patient presented with painless vaginal bleeding and placenta
covering the internal os , Which of the following is considerd a risk
factor for her presentation ?
A. HTN
B. Multigestation
C. Smoking
D. Dm

Correct answer is: B. multigestation

Multigestation is considered a risk factor for the patient’s presentation.


Explanation:
Placenta previa is a condition when the placenta implants in the lower
part of the uterus and covers the internal cervical os, which can cause
painless vaginal bleeding. Some of the known risk factors associated
with placenta previa include advanced maternal age, previous cesarean
delivery, multiparity, and smoking. However, among the given options,
multigestation (option B) or carrying multiple pregnancies, is considered
a risk factor for the patient’s presentation with painless vaginal bleeding
due to placenta previa. Hypertension (option A), smoking (option C), and
diabetes mellitus (option D) are also significant risk factors for devel-
oping placenta previa, but they are not the most crucial risk factor for
this specific clinical presentation. Therefore, option B, multigestation, is
most likely to be the risk factor for this patient’s presentation.

Dr.Ali Balharith MD, MACP, FRCPC 99


Q102- A pregnant lady came to hospital with broken arm after her
husband beat her, whats the most likely time increase violence?
A. After retirement
B. After family visit
C. Holidays
D. At work

Correct answer is: C. Holidays

Studies have shown that periods of increased stress, such as vacations,


holidays, and financial difficulties, can dramatically increase the inci-
dence and severity of domestic violence in pregnant women. Stress re-
lated to family events may also trigger the onset or possible escalation of
domestic violence.
Option A: After retirement may increase the risk of domestic violence
in some cases, but it is unrelated to the present situation of a pregnant
woman with a broken arm after being beaten by her husband.
Option B: Visitation by family members may not have a significant as-
sociation with an increase in domestic violence incidents in pregnant
women.
Option D: Violence at work has been documented but is not the most
likely time to see an increase in domestic violence in pregnant women.
Therefore, the most likely time for an increase in domestic violence
against pregnant women in this case scenario is during holidays. It is
important to identify risks and provide support to women experiencing
domestic violence, especially during such heightened stress periods.

Dr.Ali Balharith MD, MACP, FRCPC 100


Q103- A 41 week pregnancy primi came for follow up.. NST .normal
study no maternal comorbidity, what is the next step ?
A. induction of labour by oxytocin
B. induction of labour by Arm
C. follow up in 7 days

Correct answer is: C. Follow up in 7 days.

At 41 weeks of gestation, it is expected that the patient may go into la-


bor any day now. As the fetal test results are normal, and the patient has
no maternal comorbidities, there is no immediate need for induction
of labor. Based on the current guidelines, if there are no maternal or fe-
tal complications, the patient can be reassessed in seven days for fetal
well-being, cervical exam, and discussed the possibility of induction if
labor hasn’t started by that time.
Induction of labor should be reserved for cases where there is a medical
indication, such as maternal hypertension, placental insufficiency, oligo-
hydramnios, or intrauterine growth restriction. It also involves informed
consent of the patient.
Oxytocin induction (Option A) and induction with an artificial rupture
of membranes (ARM) (Option B), both are methods of induction of labor
and should be performed only with a medical indication. It is not recom-
mended under these circumstances.

Dr.Ali Balharith MD, MACP, FRCPC 101


Q104- Pregnant Female GA 37 weeks with bicornate uterus, com-
plains of subcostal heaviness, fetal kicks appreciated mainly in the
lower abdomen, leopold maneuver showed global soft part with
sluggish ballottment by first and second pelvic grip, fetal heart
sound detected at the level of maternal umbilicus, what is the most
likely presentation?
A. Face
B. Shoulder
C. Brow
D. Breech

Correct answer is: D. Breech

Breech is the most likely presentation in this case.


Explanation:
A bicornuate uterus is a congenital uterine anomaly where the uterus is
divided into two separate horns, and it increases the risk of malpresen-
tation during pregnancy. In this case, the subcostal heaviness and fetal
kicks appreciated mainly in the lower abdomen suggest that the present-
ing part is not cephalic or not positioned head-down as it is most often
the case in the last few weeks of pregnancy.
The Leopold maneuver, a standardized technique for abdominal palpa-
tion during pregnancy, showed a global soft part with sluggish ballott-
ment by the first and second pelvic grip, which is consistent with the
presence of a breech presentation. Additionally, the fetal heart sound
detected at the level of maternal umbilicus is also typical of a breech
presentation.
Breech presentation occurs when the fetal buttocks or feet are positioned
to deliver first instead of the head. It is associated with an increased risk
of complications during delivery, such as fetal distress or umbilical cord
prolapse. Therefore, it is essential to diagnose a breech presentation as
early as possible and plan accordingly for the mode of delivery.
In summary, given the clinical presentation of a pregnant female at 37
weeks gestation with a bicornuate uterus, subcostal heaviness, fetal kicks
Dr.Ali Balharith MD, MACP, FRCPC 102
mainly in the lower abdomen, global soft part with sluggish ballottment,
and the presence of fetal heart sound at the level of maternal umbilicus,
a breech presentation (option D) is the most likely presentation.
D. Biliary atresia is the most likely cause of yellowish discoloration that is
not faded, starting on the 3rd day, and not responding to phototherapy.

Explanation:

Neonatal jaundice is common in newborns, and it usually appears after


the first 24 hours of life, peaking at 3-5 days and starting to fade by the
7th day. Physiologic jaundice is a benign condition that occurs in nearly
60% of newborns and is usually self-limited. However, if jaundice per-
sists for more than two weeks, or is present at high levels on the 3rd day
of life, further evaluation and treatment are needed.
Biliary atresia is a serious medical condition in which there is a block-
age or absence of the bile ducts that causes a build-up of bilirubin in the
blood and subsequent jaundice. The onset of jaundice in biliary atresia
is usually within the first 2-4 weeks of life, but it can occur as early as
the first few days. Unlike in physiological jaundice, which resolves with
phototherapy, the jaundice caused by biliary atresia does not respond to
phototherapy and requires further evaluation and treatment.
ABO incompatibility (option A) and Rh incompatibility (option B) can
also cause neonatal jaundice, but they typically present within the first
24 hours of life. G6PD (option C) is a genetic condition that can cause
jaundice in newborns, but it is usually self-limited and not associated
with prolonged or persistent jaundice.
In summary, the most likely cause of yellowish discoloration that is not
faded, starting on the 3rd day, and not responding to phototherapy is D.
Biliary atresia which requires further evaluation and treatment.

Dr.Ali Balharith MD, MACP, FRCPC 103


Q105- Primigravida , delivers healthy baby with uneventful vaginal
delivery. After delivery of the placenta, she’s found to have lost 1000
cc of blood. She looks well, stable and her uterus is firm. What is the
most likely cause for bleeding?
A. Uterine atony
B. Coaglopathy
C. Retained products of conception
D. Genital tract laceration

Correct answer is:A. Uterine atony

The most likely cause of bleeding in this case scenario is uterine atony,
defined as the lack of normal uterine muscle tone and failure of the uter-
us to contract adequately after childbirth.
Postpartum hemorrhage (PPH) is one of the most common causes of
maternal morbidity and mortality worldwide, with uterine atony being
responsible for up to 80% of all cases. It could occur due to a variety of
risk factors, including prolonged labor, multiple gestations, macrosomia,
polyhydramnios, and previous PPH history.
Option B: Coagulopathy, or deficiencies in clotting factors, could also
cause PPH, but typically lead to more severe bleeding and are less likely
in an uneventful delivery as described in the case scenario.
Option C: Retained products of conception could be another possible
cause of PPH, but this would typically manifest with symptoms such as
fever, abdominal pain, or vaginal discharge, which are not mentioned in
this case scenario.
Option D: Genital tract laceration could also cause PPH, but is less com-
mon than uterine atony as the cause of PPH.
Therefore, based on the scenario described, the most likely cause of
bleeding is uterine atony, and prompt management with uterine mas-
sage and/or medication to contract the uterus is essential to prevent po-
tentially life-threatening complications.

Dr.Ali Balharith MD, MACP, FRCPC 104


Q106- A 70 year old female complaining of dysuria, frequency, ur-
gency. On examination there is a Grade 2 anterior wall and vaginal
prolapse. What is the most appropriate next step in management?
A. Vaginal hysterectomy
B. Anterior repair
C. Urinalysis

Correct answer is: C. Urinalysis

Urinalysis is the most appropriate next step in management for a 70s


female with dysuria, frequency, urgency, and Grade 2 anterior wall and
vaginal prolapse.
Dysuria, frequency, and urgency are all common symptoms of a urinary
tract infection (UTI), which could be a more likely etiology in this pop-
ulation. UTIs are common among older women, and a vaginal prolapse
could further increase the risk of developing a UTI. Therefore, the initial
step in evaluating a patient with suspected UTI and prolapse would be
to perform a urinalysis or urine culture to diagnose and guide antibiotic
therapy.
Option A: Vaginal hysterectomy might not be necessary or indicated in
this case, as the patient’s primary symptoms are likely secondary to an
underlying UTI.
Option B: An anterior repair could be an option later on, depending on
the size and type of prolapse, but it is not the appropriate primary inter-
vention to address a UTI.
Therefore, the appropriate initial management would be to perform uri-
nalysis to rule out UTI or guide appropriate antibiotic therapy.
C - Screening for abdominal aortic aneurysm (AAA) would be the most
important screening test for a 65-year-old male who is a heavy smoker
coming to the preventive medicine clinic.
Abdominal aortic aneurysm (AAA) is a life-threatening condition that is
more common in males, especially those with a history of smoking. The
risk of AAA increases with age, and screening for AAA is recommended
for males over 65 years who have ever smoked. AAA screening typically
Dr.Ali Balharith MD, MACP, FRCPC 105
involves an abdominal ultrasound or a CT scan, which can identify an
enlargement or ballooning of the aortic artery in the abdomen before it
ruptures.
Option A: Screening for osteoporosis is important for postmenopausal
women and men over the age of 70, or those with risk factors for osteo-
porosis. However, this would not be the most important screening test
for a heavy smoking male over the age of 65.
Option B: While colon cancer screening is important for males and fe-
males over 45, and high-risk individuals may require earlier screening, it
would not be the most important screening test for a 65-year-old male
who is a heavy smoker.
Therefore, the most important screening test for a 65-year-old male who
is a heavy smoker would be screening for abdominal aortic aneurysm
(AAA).

Q107- Female pregnant first trimester, which one of the following


blood chemistry will mostly appear ?
A. Increase in plasma Na
B. Decrease in creatinine
C. Increase BUN
D. Unchanged BUN

Correct answer is: D. Unchanged BUN.

Explanation:

During the first trimester of pregnancy, the body undergoes various hor-
monal changes that can cause some alterations in blood chemistry. How-
ever, the blood urea nitrogen (BUN) level, which is produced by the liver
and excreted by the kidneys, remains unchanged during pregnancy. It is
because the glomerular filtration rate (GFR), which is the rate at which
the kidneys filter the blood, increases during pregnancy. This increased
GFR helps the kidneys excrete more waste products efficiently, leading
to a stable BUN level.
Dr.Ali Balharith MD, MACP, FRCPC 106
Option A is incorrect as plasma sodium concentration may decrease due
to hemodilution caused by increased blood volume during pregnancy.
Option B is incorrect because creatinine clearance decreases during the
first trimester of pregnancy, leading to an increase in serum creatinine
level.
Option C is incorrect as BUN may remain unchanged during pregnancy
due to increased GFR and efficient kidney function.

Q108- Female patient complaing of menstrual irregularity,hyper-


tension, Hirtusm With high androgen in the blood US shows multi-
ple small cysts in the ovaries What is the most likely diagnosis?
A. Gynecomastia
B. Kallmann syndromes
C. klinefelter syndrome
D. Stein-Leventhal syndrome

Correct answer is: D. Stein-Leventhal syndrome

Explanation:

The patient in this case is presenting with menstrual irregularity, hyper-


tension, hirsutism (excessive hair growth in women in a male pattern),
and high levels of androgens in the blood. The presence of small cysts in
the ovaries, as seen on the ultrasound, is consistent with the diagnosis of
PCOS.
PCOS is a common hormonal disorder that affects women of reproduc-
tive age. It is characterized by the presence of multiple small cysts in the
ovaries that often lead to hormonal imbalances, including elevated levels
of androgens such as testosterone, and can lead to the signs and symp-
toms seen in this patient.
Option A - Gynecomastia, the development of breast tissue in males, is
not associated with the symptoms described in this case and is therefore
an incorrect answer.
Option B - Kallmann syndrome is a genetic disorder that affects the abil-
Dr.Ali Balharith MD, MACP, FRCPC 107
ity to smell and leads to delayed or absent puberty, and is not associated
with the symptoms in this case, and so is also an incorrect answer.
Option C - Klinefelter syndrome is a genetic disorder where affected
individuals have an extra X chromosome, leading to male hypogonadism
and infertility. It is not associated with the typical hormonal imbalances
seen in PCOS or the symptoms described in this case, and is therefore
also an incorrect answer.
Therefore, the most likely diagnosis in this case is D - Stein-Leventhal
syndrome or PCOS.

Q109- Pregnant with recurrent UTI what to do?


A. X ray
B. US
C. Ureteroscopy
D. Cystoscopy and RET’s

Correct answer is: B. US

In a pregnant woman with recurrent UTI, the appropriate diagnostic and


management options can depend on the severity of the symptoms and
the gestational age of the pregnancy. In general, antibiotics are the pri-
mary treatment for UTI in pregnancy.
Option A - X-ray is not usually indicated in the diagnosis of UTI, particu-
larly in pregnant women, as radiation exposure may harm the develop-
ing fetus.
Option B - An ultrasound (US) is often the first-line imaging modality
used to evaluate UTI in pregnancy as it is non-invasive and doesn’t ex-
pose the mother or fetus to ionizing radiation. Ultrasound can help iden-
tify anatomic abnormalities that predispose women to UTIs, such as uri-
nary tract obstruction or vesicoureteral reflux.
Option C - Ureteroscopy is an endoscopic procedure that involves pass-
ing a thin, flexible scope with a camera attached through the urethra and
bladder and up into the ureters and kidneys. While ureteroscopy can be
helpful in the diagnosis of UTI, it is an invasive procedure that carries
Dr.Ali Balharith MD, MACP, FRCPC 108
risks in the pregnant population.
Option D - Cystoscopy and retrograde ureteral catheterization (RET’s)
is an invasive diagnostic procedure that involves introducing a flexi-
ble scope into the bladder to visualize the urethra, bladder, and ureter-
al openings. This procedure is not typically used as a diagnostic tool in
pregnant women.
Therefore, the most appropriate diagnostic option for a pregnant wom-
an with recurrent UTI is B - ultrasound. However, the management plan
should be discussed with the obstetrician and the treating physician to
ensure proper evaluation and care. Additionally, proper prevention strat-
egies, such as proper hygiene, additional fluid intake, or other prophylac-
tic antibiotics, should be considered to prevent recurrent UTIs.

Q110- A 50 year old female with Chronic lower abdominal pain and
bleeding Imaging shows: fibroid What’s the most appropriate man-
agement?
A. Hysterectomy
B. Combined OCP
C. Myomectomy
D. Dilation and curettage

Correct answer is:A. Hysterectomy

The most appropriate management for a 50-year-old female with chron-


ic lower abdominal pain and bleeding with imaging showing a fibroid de-
pends on the severity of symptoms, the size and location of the fibroid,
and the patient’s desire for fertility.
If the fibroid is causing severe symptoms and affecting the patient’s qual-
ity of life, the most appropriate management would be A - Hysterecto-
my, which is the surgical removal of the uterus. This is a definitive treat-
ment option that eliminates the possibility of future fibroid growth and
recurrence. However, this option is not suitable for patients who desire
to preserve their fertility.
Dr.Ali Balharith MD, MACP, FRCPC 109
If the patient desires to preserve their fertility or if the fibroid is smaller
in size and not causing severe symptoms, the most appropriate manage-
ment option would be C - Myomectomy, which is the surgical removal
of the fibroid while preserving the uterus. This option is suitable for pa-
tients who wish to maintain their fertility potential and avoid a hyster-
ectomy.
Option B - Combined OCP (oral contraceptive pills) may be used to alle-
viate symptoms of fibroids, such as heavy menstrual bleeding and pelvic
pain. However, OCPs do not treat the underlying fibroid and may not be
effective for all patients.
Option D - Dilation and curettage (D&C) may be used to diagnose and
treat abnormal uterine bleeding, but it is not a treatment option for fi-
broids.
Therefore, the most appropriate management for a 50-year-old female
with chronic lower abdominal pain and bleeding with an imaging find-
ing of a fibroid would be A - Hysterectomy if the fibroid is causing severe
symptoms and affecting the patient’s quality of life, or C - Myomectomy
if the patient desires to preserve their fertility or if the fibroid is smaller
in size and not causing severe symptoms.

Q111- A 38 weeks GA, sever headaches, LL oedema, mildly elevated


LFT & Low platelets 45 What to do ?
A. C/S
B. Induction of labor
C. Observe

Correct answer is: A. C/S

Based on the given information, the most appropriate management for


the patient’s condition is A- C/S, which means caesarean section.
Explanation:
The clinical presentation of severe headaches, lower limb edema, elevat-
ed liver function tests, and a low platelet count characterizes the classi-
Dr.Ali Balharith MD, MACP, FRCPC 110
cal features of preeclampsia, a severe complication of pregnancy. Timely
intervention is essential to avoiding severe maternal and fetal complica-
tions, which includes delivering the baby.
Option B- Induction of labor is not appropriate for this patient because
induction of labor is contraindicated in women with severe preeclamp-
sia due to a high risk of maternal or fetal complications.
Option C- Observe is not recommended for this situation because pro-
gressive worsening of the patient’s condition, such as development of
eclampsia (seizures), can occur, and immediate intervention may be re-
quired to safeguard the health of both the mother and baby.
In summary, if a pregnant woman presents with preeclampsia features,
as presented in the scenario, the definitive management is prompt deliv-
ery by C/S or induction of labor, depending on the clinical circumstanc-
es.

Q112- Uncontrollable Vaginal bleeding, developed DVT, what is the


Dx ?
A. Uterine rupture
B. Placenta rupture
C. placenta previa accreta

Correct answer is:C. Placenta previa accreta

Based on the given scenario of uncontrollable vaginal bleeding and DVT


development during pregnancy likely diagnosis is C - Placenta previa ac-
creta.

Explanation:
Placenta previa accreta is a pregnancy-related complication where the
placenta abnormally attaches to the uterine wall leading to severe and
persistent vaginal bleeding, putting both mother and fetus at high risk
for significant complications.. In this condition, there is a greater chance
for the development of deep vein thrombosis (DVT) due to blood clots.
Dr.Ali Balharith MD, MACP, FRCPC 111
Uterine rupture (Option A) is an uncommon obstetric emergency that
occurs typically during labor, which can result in vaginal bleeding. How-
ever, it is less likely the cause of the bleeding and DVT in the scenario
described.
Placental abruption (Option B) is another possible cause of vaginal bleed-
ing and DVT, but typically occurs earlier in pregnancy than the given
scenario. It occurs when the placenta separates from the wall before the
delivery of the baby.
In summary, with the presentation of uncontrollable vaginal bleeding
coupled with DVT development during pregnancy, placenta previa ac-
creta could be the likely diagnosis, and urgent intervention by an obste-
trician would be required to ensure both mother and fetus’s safety and
health.

Q113- Vertex presentations.. How to diver ?


A. Vacuum
B. CS
C. Forcebs
D. NVD

Correct answer is: D. NVD (Normal Vaginal Delivery).

The method of delivery with vertex presentation (head-first) of the fetus


can depend on many factors, such as the stage of labor, maternal and
fetal health, and other factors. However, in general, the most common
approach for delivery in a vertex presentation is D- NVD (Normal Vaginal
Delivery).
Explanation:
A vertex presentation is considered the normal presentation for delivery.
In this presentation, the baby is oriented head-first, meaning that the
head is the largest part of the fetal body and comes first.
For a NVD, the mother can push the baby through the birth canal during
contractions, and the doctor or midwife will help guide the head and
Dr.Ali Balharith MD, MACP, FRCPC 112
shoulders of the newborn through the birth canal, while monitoring
both maternal and fetal health. If the baby is healthy, the presentation is
normal, and the delivery is progressing well, NVD is a safe and common
method of delivery.
In some cases, complications during labor, maternal or fetal distress, or
other factors may require the use of other interventions such as assisted
delivery using a vacuum (Option A) or forceps (Option C), or in some
cases, a cesarean section (Option B) may be necessary.
In summary, a normal vertex presentation can often result in a normal
vaginal delivery, but the delivery method ultimately depends on many
maternal, fetal, and other clinical factors.

Q114- Patient with Amenorrhea( LH : 37 FSH : 50 “luteal phase” the


cause of Amenorrhea ?
A. Ovarian dysfunction
B. Asherman Syndrome

Correct answer is:A. Ovarian dysfunction

In the given scenario of a patient with amenorrhea and high LH and FSH
levels during the luteal phase, the likely cause of amenorrhea is A - Ovar-
ian dysfunction.

Explanation:
Amenorrhea refers to an absence of menstruation in women of repro-
ductive age, which may be indicative of various underlying causes. As
mentioned in the scenario, the patient has elevated FSH and LH levels,
which suggests ovarian dysfunction. FSH (Follicle-stimulating hormone)
and LH (Luteinizing hormone) are hormones produced by the brain that
regulate the menstrual cycle. High levels of LH and FSH during the lute-
al phase can indicate a lower-than-normal level of estrogen production
by the ovaries.
Ovarian dysfunction can occur due to various causes such as polycystic
ovary syndrome or premature ovarian failure (early menopause). These
Dr.Ali Balharith MD, MACP, FRCPC 113
conditions can prevent the ovaries from functioning normally, leading
to a diminished production of estrogen, a leading cause of amenorrhea.
Asherman Syndrome (Option B) is a rare condition that is character-
ized by the formation of scar tissue inside the uterus, leading to men-
strual abnormalities such as reduced menstrual flow or amenorrhea.
However, this condition is not commonly associated with elevated lev-
els of LH and FSH.

Q115-48 female presented with heavy vagainal bleeding and in us


there is clots blood in uterus the endometrium thinking it was
13mm with two 1mm lesions, endometrial sampling it was negative
what is the best action to reach the diagnosis
A. Sonohysterogram
B. Ct abdomen and pelvsi
C. Mri pelvis
D. Speculam examination

Correct answer is: A. Sonohysterogram.

A sonohysterogram is a procedure that uses ultrasound to visualize the


uterus and the lining of the uterus (endometrium). It is a minimally inva-
sive procedure that can be done in the doctor’s office.
In this case, the patient has heavy vaginal bleeding and a thickened en-
dometrium. The endometrial sampling was negative, which means that
there was no cancer found. However, the negative result does not rule
out the possibility of cancer. A sonohysterogram can help to visualize
the endometrium and look for any abnormalities that may be missed on
a regular ultrasound.
The other options are not as appropriate in this case.
* *CT abdomen and pelvis:* A CT scan is a more invasive procedure than
a sonohysterogram and is not necessary in this case.
* *MRI pelvis:* An MRI is a more expensive procedure than a sonohys-
terogram and is not necessary in this case.
* *Speculam examination:* A speculam examination is a visual inspec
Dr.Ali Balharith MD, MACP, FRCPC 114
tion of the vagina and cervix. It is not as sensitive as a sonohysterogram
for detecting abnormalities in the endometrium.

Q116- What is the time interval between ovulation and cleavage in dicho-
rionic diagnostic twins?
A. 0-72 h
B. 4-8 days
C. 9-12 days
D. >12 days

Correct answer is: B. 4-8 days.

Dichorionicity is the condition of having two separate placentas in a twin


pregnancy. This can occur if the fertilized egg splits into two separate
embryos within 4-8 days after fertilization. If the fertilized egg splits af-
ter 8 days, the twins will share a placenta and be monochorionic.
The time interval between ovulation and cleavage in dichorionic diag-
nostic twins is therefore 4-8 days.
Here are the reasons why the other options are wrong:
* *A. 0-72 h:* This is the time interval between ovulation and fertiliza-
tion. Fertilization occurs when the sperm fertilizes the egg. The time
interval between ovulation and cleavage is longer, as it takes time for the
fertilized egg to divide into two separate embryos.
* *C. 9-12 days:* This is the time interval between fertilization and im-
plantation. Implantation occurs when the fertilized egg attaches to the
lining of the uterus. The time interval between ovulation and cleavage is
earlier, as it takes time for the fertilized egg to divide into two separate
embryos.
* *D. >12 days:* This is the time interval between implantation and the
start of pregnancy. The time interval between ovulation and cleavage is
earlier, as it takes time for the fertilized egg to divide into two separate
embryos.
Dr.Ali Balharith MD, MACP, FRCPC 115
Dr.Ali Balharith MD, MACP, FRCPC 116

You might also like

pFad - Phonifier reborn

Pfad - The Proxy pFad of © 2024 Garber Painting. All rights reserved.

Note: This service is not intended for secure transactions such as banking, social media, email, or purchasing. Use at your own risk. We assume no liability whatsoever for broken pages.


Alternative Proxies:

Alternative Proxy

pFad Proxy

pFad v3 Proxy

pFad v4 Proxy